You are on page 1of 53

BREAK UP OF QUESTIONS

UNIT Topic No. of Questions


India Foreign Policy 1
India and Neighborhood 6
India and Extended Neighborhood 3
India and Major Powers-USA, Russia, China, Etc. 5
India and other Bilateral Relations 2
India and Groupings 10
1. International
Rigional Groupings 10
Relations
International Organisations 10
Miscellaneous Topics 10
Indices and Reports 5
Places in News 8
Exercises and Operations 5
Total 75
Polity and Governance 8
Economy 10
Geography, Environment, places in news 7
History and culture 3
2. Regional
Programmes and Schmes 22
Personalities in news 5
Awards and Reports 10
Miscellaneous 10
Total 75
GRAND TOTAL 150

Format of questions
QUESTIONS
Regional International Total
Direct Questions 49 43 92
Two Statement Based 14 13 27
Three Statement Based 8 10 18
Four Statement Based 4 6 10
Five Statement Based - 2 2
Six Statement Based - 1 1
Total 75 75 150

AMIGOS IAS 1
APPSC – GROUP-II, Prelims – TEST 12 (01-02-2024)
1. Recently, the Indian Defence Minister and his (3) The project is also funded by France.
counterpart laid the “Ekatha Harbour” (4) The first three STAR-C centres are being
foundation on his foreign tour to set up in India, Morocco, and South Africa.
(1) Maldives (2) Sri Lanka Answer: 2
(3) Mauritius (4) Seychelles Explanation:
Answer: 1 • India is considering expanding its Solar
Explanation: Technology and Application Resource
Ekatha Harbour: Centres (STAR -C) initiative to a number of
• The Indian Defence Minister and his Pacific Island countries.
Maldivian counterpart laid the “Ekatha About STAR-C Initiative:
Harbour” foundation. • The program aims to boost solar power
• Ekatha Harbour is being constructed in the ecosystems in the poorest countries.
Maldives for the Maldives National • The initiative is run by the International
Defence Forces Coast Guard. Solar Alliance in partnership with the
• The Harbour is being constructed at United Nations Industrial Development
Sifavaru in Uthuru Thila Falhu (UTF) atoll. Organisation (UNIDO). Hence, Option 2 is
• India is assisting in the construction of the incorrect.
naval dockyard, and it is regarded as one • The first three STAR-C centres are being
of India's most significant grant-in-aid set up in India, Morocco, and South Africa.
projects. • The centres are expected to be
• The Indian Defence Minister also gave the operational by 2024.
Maldives National Defence Forces Coast • It aims to create a strong network of
Guard an additional landing craft in line institutional capacities within ISA member
with India’s vision of security and growth states to enhance quality infrastructure
for all in the region (SAGAR). for the uptake of solar energy products
• Maldives is said to be one of India’s crucial and services.
maritime neighbors in the Indian Ocean • The project is also funded by France.
Region (IOR), and the Indian government Programme’s objectives:
is looking to expand ties with Male in • Building solar workforces, standardizing
areas of defence and security amid China’s products, setting up infrastructure, and
efforts to expand its influence in the raising awareness among policymakers in
region. developing countries.
• The Ministers have agreed to explore • Through this initiative, India aims to
additional avenues for cooperation, enhance the institutional capacities of
including in defence trade, capacity International Solar Alliance member
building and joint exercises. states and contribute to the development
2. Which of the following statements is of quality infrastructure for solar energy
incorrect with respect to the “Solar uptake.
Technology and Application Resource 3. “Zangnam”, often in the news, is a bone of
Centers (STAR-C)" initiative?” contention between
(1) The programme aims to boost solar power (1) India and China (2) India and Pakistan
ecosystems in the poorest countries. (3) India and Sri Lanka (4) India and Maldives
(2) The initiative is run by the International Answer: 1
Solar Alliance in partnership with the
Explanation:
United Nations Development Program
• China’s move to rename eleven places in
(UNDP).
Arunachal Pradesh.

AMIGOS IAS 2
• China has renamed eleven places in India’s • Controversy: Sending cluster munitions
Arunachal Pradesh, which they consider from the U.S. to Ukraine sparks global
to be “Zangnam” or “South Tibet” (in debate.
English). History of Cluster Munitions:
• It is claimed that the move to • Origin: Used since WWII; notable
“standardise” names in Zangnam instances of use in Vietnam, Afghanistan,
“completely falls within China’s and Iraq.
sovereignty”. • Toll: Cluster munitions linked to 56,000 to
4. “Junta Regime," often seen in news, is 86,000 deaths in affected areas since the
associated with which of the following 1960s.
countries? Convention on Cluster Munitions and its
(1) Niger (2) Myanmar Members:
(3) Pakistan (4) Bangladesh • Creation: The Convention on Cluster
Answer: 2 Munitions (CCM) was established in May
Explanation: 2008.
• A junta regime is composed of military • Signatories: 112 countries, including
officers who seize ruling power in a NATO nations like Canada, Germany,
country. It may be seen as a subset of France, and the UK.
military rule, as the primary institution • Non-Signatories: Key players like the U.S.,
driving a junta government is the military. Russia, China, Israel, and India haven’t
• The junta government led by military joined.
leader Min Aung Hlaing of Myanmar is a • Obligations: CCM bans use, production,
classic example. stockpiling, and transfer; mandates victim
• The Myanmar junta usurped power assistance.
through a coup in February 2021, refusing 6. What was theme of the Andhra Pradesh
to recognise the elections in which former government’s two-day Global Investors
State Counsellor Aung San Suu Kyi’s Summit 2023, held on March 3 and 4 at
National League for Democracy (NLD) had Visakhapatnam?
a landslide victory. (1) Advantage Andhra Pradesh-Where
5. Which of the following countries is a Fortune Meets Prosperity.
signatory to the “Convention on Cluster (2) Advantage Andhra Pradesh-Where
Munitions”? Abundance Meets Prosperity.
(1) Germany (2) USA (3) Sun Rise Andhra Pradesh- Where
(3) Russia (4) India Abundance Meets Prosperity.
Answer: 1 (4) Advantage Andhra Pradesh-Where
Explanation: Investments progress Prosperity.
Context: Answer: 2
 “Amidst the U.S. sending cluster Explanation:
munitions to aid Ukraine, explore legality, • Andhra Pradesh government conducted a
history, and humanitarian concerns of two-day Global Investors Summit 2023 on
these weapons.” March 3 and 4 at Visakhapatnam.
About Cluster Munitions: • The theme of the summit is "Advantage
• Definition: Cluster munitions, also known Andhra Pradesh – Where Abundance
as cluster bombs, release multiple Meets Prosperity.
bomblets upon deployment. • The CM said the state government had
• Impact: Bomblets explode on impact or formed a monitoring committee to ensure
remain dormant, posing ongoing dangers that the MoUs, worth Rs 13 lakh crore,
to civilians. resulted in the starting of units.
Commensurate with the overwhelmingly

AMIGOS IAS 2
positive business sentiment, we have 8. Who is the current executive chairman of the
executed 352 MOUs with an investment State Legal Services Authority of Andhra
commitment of over Rs 13,05,663 crore Pradesh?
and employment potential of over (1) Dhiraj Singh Thakur
6,03,223 persons. (2) Akula Venkata Sesha Sai
• In the energy sector alone, 40 MoUs have (3) Upmaka Durga Prasad Rao
been signed worth Rs 8,84,823 crore of (4) Cheekati Manavendranath Roy
investment that will create employment
Answer: 2
for 1,90,268 people. In the IT and ITES
Explanation:
sectors, 56 MoUs have been signed,
amounting to Rs 25,587 crore of • The executive chairman of the State Legal
investment and generating employment Services Authority and Andhra Pradesh
for 1,04,442 persons. High Court judge Justice Akula Venkata
Sesha Sai said on May 7 that early justice
• In the tourism sector, 117 MoUs have
could be possible through the legal service
been signed for an investment to the tune
authority. He was inaugurating the Nyaya
of Rs 22,096 crore that will create
Seva Sadan Bhavan in
employment for 30,787 people.
Rajamahendravaram city.
Source:
Source:
https://indianexpress.com/article/busine
https://www.thehindu.com/news/nation
ss/ap-govt-concludes-2-day-global-
al/andhra-pradesh/early-justice-possible-
investors-summit-8479763/
with-legal-services-authority-high-court-
7. The Department of Agriculture in Andhra
judge/article66823859.ece
Pradesh won the Skoch Award (silver) in 2023
9. Adhra Pradesh assigned lands (prohibition of
for its initiative in promoting?
transfers) (amendment) ordinance, 2023,
(1) Rythu Bharosa Kendralu
permitted to sell the assigned land after a
(2) e-crop system period of
(3) Community Managed Natural Farming (1) 5 years (2) 10 years
(4) Polambadi Program (3) 15 years (4) 20 years
Answer: 2 Answer: 4
Explanation: Explanation:
• The department of agriculture in Andhra • AP Assigned Lands (Prohibition of
Pradesh won the Skoch Award (silver) for Transfers) Act, 1977 is a protective
its initiative in promoting e-crop legislation and it prohibits the transfer of
applications for the year 2023. The awards lands assigned to landless poor persons
were presented in a programme held in for the purpose of cultivation and
Delhi. provides for restoration of such
• The special commissioner said that the transferred lands to the original assignee
award was presented to AP for its and also for punishment to those
initiative in using digital Android acquiring such lands.
technology for an e-crop system to • The condition of non-alienability of
identify farmers on the field and register assigned lands impacted the poorest of
crops. The e-crop details are displayed on the poor by preventing them from making
notice boards at Gram Sabhas. The e-crop use of the same for emergent family
system will help to get crop information needs.
on a single platform.
• AP government constituted a committee
Source: under the chairmanship revenue minister
https://www.thehansindia.com/andhra- to examine the issue of the right to sell the
pradesh/prakasam-police-receive-skoch- assigned lands for the welfare and
award-778514?infinitescroll=1 financial ease of the assignees.

AMIGOS IAS 3
• The committee, after examining the of East Godavari, the Uppada Jamdani
relevant revenue laws enforced in the sarees of Kakinada, the frozen shrimp of
neighboring states and the ground Bapatla, the Bobbili Veena of
situation in the state, recommended inter Vizianagaram, the Udayagiri wooden
alia that the original assignee or his/her cutlery of Nellore, and the coconut and
legal heir may be permitted to alienate the coir products of BR Ambedkar Konaseema
assigned land after a period of 20 years district.
from the date of assignment as per the • It is shrimp in Visakhapatnam, handloom
procedure prescribed and without silk sarees in Kurnool, Madanapalle silk
reference to the government. sarees in Annamayya, Araku Coffee in
• The promulgation of the AP assigned lands Alluri Sitarama Raju district, silk sarees in
(prohibition of transfers) (amendment) Sri Satya Sai, Ponduru cotton sarees in
ordinance, 2023 (AP ordinance 9 of 2023) Srikakulam and Venkatagiri handlooms in
by the governor on July 27, 2023. Tirupati.
• With the passing of this bill, the assigned Source:
landowners would get full rights on their https://www.deccanchronicle.com/natio
lands if they have completed 20 years. n/current-affairs/191023/14-andhra-
Source: pradesh-products-shortlisted-for-odop-
https://www.deccanchronicle.com/natio awards-2023-24.html
n/politics/260923/grant-of-rights-to- 11. Which of the following countries has not
assigned-lands-beneficiaries-historical- signed Free Trade Agreements (FTAs) with
other-stat.html India till date?
10. Which of the following products are matched (1) Australia (2) Singapore
correctly with their district of origin? (3) United Kingdom (4) Japan
I. Uppada Jamdani sarees: Kakinada Answer: 3
II. Udayagiri wooden cutlery: Nellore Explanation:
III. Coir products: BR Ambedkar Konaseema.  India has signed 13 Free Trade
Select the correct answer from the options Agreements (FTAs) – Sri Lanka, SAFTA,
given below. Nepal, Bhutan, Thailand, Singapore,
(1) Only I and II (2) Only II and III ASEAN, South Korea, Japan, Malaysia,
(3) Only I and III (4) All the above Mauritius, UAE, Australia.
Answer: 4 12. India made its first-ever payment for oil in
Explanation: rupees to which of the following countries?
• One District, One Product (ODOP) (1) Saudi Arabia (2) Qatar
programme, unveiled by the Department (3) USA (4) UAE
for Promotion of Industry and Internal Answer: 4
Trade (DPIIT). This was aimed at fostering Explanation:
the socio-economic development of all • India, the world’s third-largest energy
the districts across the country. consumer, made its first-ever payment in
• Andhra Pradesh has submitted 26 rupees for crude oil purchased from the
applications from fields like textiles, United Arab Emirates (UAE), signalling
handicrafts, handlooms, fisheries, and what could be a strategic push to promote
industrial products for the One District, the local currency globally.
One Product (ODOP) Awards panel. Of • India and the United Arab Emirates (UAE)
these, 14 products from AP are among the have signed a pact to establish a
64 shortlisted for the next stage of framework to promote the use of the
selection. rupee and UAE Dirham (AED) for cross-
• The 14 products of AP were the border transactions.
Mangalagiri Handloom Sarees of Guntur, • The MoU on establishing a framework for
the Pulagurtha clothing, shirting & sarees the use of local currencies for transactions

AMIGOS IAS 4
between India and the UAE, aims to put in 14. Consider the following pairs with respect to
place a Local Currency Settlement System India’s rescue Operations:
to promote the use of INR (Indian rupee) I. Ganga: Yemen
and AED (UAE Dirham) bilaterally. II. Kaveri: Sudan
• The framework for the use of local III. Operation Ajay: Ukraine
currencies for transactions between India Which of the above pair(s) is/are matched
and the UAE aims to put in place a Local correctly?
Currency Settlement System (LCSS). The
(1) Only I (2) Only II
creation of the LCSS would enable
(3) Only III (4) All I, II, III
exporters and importers to invoice and
pay in their respective domestic Answer: 2
currencies, which in turn would enable the Explanation:
development of an INR-AED foreign • Operation Ganga was an evacuation
exchange market. mission carried out by the Indian
• This arrangement would also promote government to rescue its citizens stranded
investments and remittances between the in neighboring countries during the 2022
two countries. The use of local currencies Russian invasion of Ukraine.
would optimize transaction costs and • Operation Kaveri is a codename for India’s
settlement time for transactions, evacuation effort to bring back its citizens
including for remittances from Indians stranded in Sudan amid intense fighting
residing in the UAE. between the army and a rival paramilitary
13. “The East Turkestan Islamic Movement force in April 2023.
(ETIM)” is a militant separatist group based • Operation Devi Shakti was an operation of
in? the Indian Armed Forces to evacuate
(1) China (2) Turkmenistan Indian citizens and foreign nationals from
(3) Russia (4) Kazakhstan Afghanistan after the collapse of the
Islamic Republic of Afghanistan and the
Answer: 1
fall of Kabul to the Taliban.
Explanation:
• India initiated “Operation Ajay” to
East Turkestan Islamic Movement (ETIM) :
evacuate its citizens from Israel.
• The East Turkestan Islamic Movement
15. “Initiative on Critical and Emerging
(ETIM) is a militant separatist group based
Technologies (iCET)” is a co-production of jet
in China’s Xinjiang region, which seeks to
engines, long-range artillery, and infantry
establish an independent state called East
between India and which of the following
Turkestan.
countries?
• The Chinese government has accused
(1) Russia (2) France
ETIM of carrying out several terrorist
(3) USA (4) Australia
attacks in Xinjiang and other parts of
China. China has also claimed that the Answer: 3
group has links with other terrorist Explanation:
organizations such as Al-Qaeda and the • Recently, India and the United States have
Taliban. taken a significant step towards
• The United Nations has designated ETIM strengthening their strategic partnership
as a terrorist organization, and several and driving technology and defense
countries, including the United States and cooperation. Under the Initiative on
the European Union, have also listed the Critical and Emerging Technologies (iCET),
group as a terrorist organization. the two nations have unveiled a roadmap
• India has expressed concern about the for enhanced collaboration in high-
activities of ETIM, particularly in the technology areas.
context of its growing ties with Pakistan- • The initiative focuses on addressing
based militant groups. regulatory barriers, aligning export

AMIGOS IAS 5
controls, and fostering deeper II. It is a designated National Geological
cooperation in critical and emerging fields. Heritage Monument Site of India by the
What is the iCET? Geological Survey of India.
• The iCET was announced by India and the Which of the above statements is/are correct?
US in May 2022 and was officially (1) Only I (2) Only II
launched in January 2023 and is being run (3) Both I and II (4) Neither I Nor II
by the National Security Council of both Answer: 2
countries. Explanation:
• Under iCET, both countries have identified • Despite its scientific and cultural
six areas of cooperation which would significance, Erra Matti Dibbalu (Red Sand
include co-development and co- Dunes), nestled between Vishakhapatnam
production, which would gradually be and Bhimunipatnam, is facing escalating
expanded to QUAD, then to NATO, threats that put its unique features at risk.
followed by Europe and the rest of the Hence, Statement I is not correct.
world.
• Designated as one of the 34 notified
• Under iCET, India is ready to share its core National Geological Heritage Monument
technologies with the US and expects Sites of India by the Geological Survey of
Washington to do the same. India in 2016, the Erra Matti Dibbalu
• It aims to promote collaboration in critical stands as a testament to Earth’s evolution,
and emerging technology areas, including providing valuable insights into its history.
AI, quantum computing, semiconductors, Hence, Statement II correct.
and wireless telecommunication. • Spanning a length of 5 km along the coast
16. The “Tiger 108 Operation” in Andhra Pradesh and nearly 2 km in width, the dunes are
is related? about 16 km from Visakhapatnam.
(1) To conserve 108 tigers in Andhra Pradesh • Erra Matti Dibbalu is one of the last three
with a state-specific plan. remaining gullied red dune formations in
(2) To trace the missing mother tiger in Southeast Asia, joining similar sites such
Nandyal District. as Teri Sands in Tamil Nadu and one more
(3) To provide toll-free number to call forest in Sri Lanka.
officials in case of any road accidents • Its rare scientific paleo-coastal red sand
involving tigers. dunes harbour mesolithic and neolithic
(4) To provide ambulance services to tigers materials, marking a first in the country.
within wildlife sanctuaries. Comprising a mixture of silt, clay, and
Answer: 2 sand, the distinct red colour arises from
Explanation: the oxidation of garnets in the earth.
• Andhra Pradesh forest department has Source:
launched 'Tiger 108' operation. https://www.newindianexpress.com/xplo
• Tiger 108 was to trace the missing mother re/2023/Dec/02/navigating-threats-to-
tiger of the four cubs found at preserve-visakhapatnams-erra-matti-
Gummadapuram village in Kothapalli dibbalu-2638169.html
mandal in Nandyal district. 18. Who is the lyricist for the song "Naatu-
Source: Naatu,” which was recently awarded the Best
https://www.thehansindia.com/andhra- Original Song Award at the 95th Academy
pradesh/nandyal-tiger-108-operation- Awards in Los Angeles, United States of
launched-to-trace-mother-tiger-786892 America?
17. Consider the following statements with (1) Rahul Sipligunj and Kaala Bhairava
reference to “Erra Matti Dibbalu (Red Sand (2) Chandrabose
Dunes)”. (3) MM Keeravani
I. They are located in East Godavari District, (4) SS Rajamouli
along the Bay of Bengal coast. Answer: 2

AMIGOS IAS 6
Explanation: Ltd. (APCCADB), which was providing Long
• The RRR film’s ‘Naatu Naatu’ soundtrack Term Investment Credit to farmers was
was awarded the Best Original Song merged with The Andhra Pradesh State
Award at the 95th Academy Awards in Los Co-operative Bank Ltd. (APCOB), with
Angeles, United States of America. effect from 30th April 1994
• The film’s ‘Naatu Naatu’ soundtrack was • Following the NABARD Consultancy
awarded the Best Original Song Award at Services (NABCONS) recommendations,
the 95th Academy Awards in Los Angeles, the government amended the 1964 law to
United States of America. strengthen cooperative banking
Artists behind the song: governance and appointed professionals
• Singers: Rahul Sipligunj and Kaala Bhairava to run APCOB and DCCBs through a
uniform selection process. We also
• Lyricist: Chandrabose
provided `295 crore as share capital for
• Composer: MM Keeravani
DCCBs.
Source:
• The AP CM unveiled APCOB’s new logo
https://www.livemint.com/news/india/n
and the APCOB Brand Identity Guidelines
aatu-naatu-from-rrr-wins-oscar-for-
(BIG) book. The bank presented him with
original-song-11678676452086.html
a dividend cheque of 55.93crore.
19. Consider the following statements with
• The AP CM said, "For the first time, APCOB
respect to the Andhra Pradesh State
is giving a dividend to the government,
Cooperative Bank Limited (APCOB).
and I am happy to receive it. This remains
I. It has completed its 50 years of inception as a special day in the state's cooperative
in 2023. history." Hence, Statement II is incorrect.
II. APCOB never made dividends to the Source1:
Government till date. https://www.deccanchronicle.com/natio
Which of the above statements is/ are not n/in-other-news/040823/apcob-giving-a-
correct? dividend-for-first-time.html
(1) Only I (2) Only II Source 2: https://apcob.org/
(3) Both I and II (4) Neither I Nor II 20. Consider the following Padmashree
Answer: 3 Awardees with their respective fields of
Explanation: eminence:
• The Andhra Pradesh State Cooperative I. Kota Satchidananda Sastry: Art
Bank Limited (APCOB), is a Scheduled II. Abbareddy Nageswara Rao: Science and
State Cooperative Bank in the state of technology
Andhra Pradesh included in the second III. Sankurathri Chandra Sekhar: Social Work
schedule of the Reserve Bank of India Act., IV. Prakash Chandra Sood: Literature and
w.e.f. 16th July 1966 and has been earning Education
profits for the past 60 years and is placed
Which of the above pairs are correct?
in ‘A’ Category consistently in Audit
(1) Only I and II (2) Only II and III
Classification.
(3) Only III and IV (4) All the above
• The Bank is committed to agricultural and
rural development through cooperatives Answer: 4
through trust, integrity and service to the Explanation:
farming community, since its inception on • Seven eminent persons from Andhra
4th August 1963 under the Andhra Pradesh are among the 91 selected for the
Pradesh Cooperative Societies (APCS) Padma Shri Awards by the Center.
Act,1949. Hence, Statement I is incorrect. • Koduri Marakatamani Keeravani,
• Under the Single Window Credit Delivery popularly known as MM Keeravani, hails
System, The Andhra Pradesh Co-operative from Kovvur in West Godavari district. He
Central Agricultural Development Bank got international recognition for the
popular ‘Naatu Naatu’ song.

AMIGOS IAS 7
• In the social work category, Sankurathri Answer: 2
Chandra Sekhar has the Padma honour. Explanation:
Chandra Sekhar, a biologist with Health • Nepal has declined China’s invitation to
Canada, settled in Ottawa. He lost his wife join President Xi Jinping’s Global Security
and two children in the bombing of an Air Initiative (GSI).
India flight. He decided to move ahead About Global Security Initiative (GSI)
and give his life a new purpose. In 1988,
• The GSI aims to contribute to global peace
he sold his house in Canada and settled
and stability by promoting fairness and
down in his wife’s village in Kakinada. He
justice among nations.
started the Sankurathri Foundation in
• It was launched in April 2022.
1989 with the aim of providing good
health facilities to rural people. • The GSI looks to counter the US Indo-
Pacific strategy and the Quad (India, US,
• In the Science and Engineering category,
Australia, and Japan grouping).
Ganesh Nagappa Krishnarajanagara has
been selected for the award. 22. With which of the following countries does
India have a 2+2 Ministerial Dialogue on
• Ganesh Nagappa, a bioorganic chemist, is
strategic and security issues?
working as the Director of the Indian
Institute of Science Education and I. United States II. Australia
Research, Sri Rama College, Tirupati. III. Japan IV. Russia
Earlier, he worked as a scientist at the Select the correct answer using the options
Centre for Cellular and Molecular Biology, given below:
Hyd, and the National Chemical (1) Only I, II, III (2) Only II, III, IV
Laboratory, Pune. He worked as a (3) Only III, IV, I (4) All I, II, III, IV
professor and director at IISER, Pune. He Answer: 4
received the Medal for Young Scientists Explanation:
from the Indian National Science
 India has 2+2 Ministerial Dialogue with the
Academy, Delhi.
US, Australia, Japan and Russia. The
• Abbareddy Nageswara Rao, a dialogue involves foreign and defense
sexagenarian scientist who discovered 33 ministers of both countries.
orchid species in Northeast India, has also
23. Consider the following Awards with their
been selected for the Padma Shri Award.
countries.
• In the arts category, Bhagavathar Kota
Country Award
Satchidananda Sastry of Guntur, a
Burrakatha exponent, and CV Raju, an I. France - Legion of Merit
Etikoppaka artist, have won Padma II. Egypt - Order of Nile
honours. III. Fiji - Companion of Order
• Prakash Chandra Sood, an honorary IV. USA - Grand Cross of the
professor at Sri Sathya Sai Institute of Legion of Honor
Higher Learning, has been selected for How many of the above pairs are correct?
Padma Shri in the Literature and (1) Only 1 pair (2) Only 2 pairs
Education category. (3) Only 3 pairs (4) All 4 pairs
Source: Answer:2
https://www.newindianexpress.com/stat Explanation:
es/andhra-pradesh/2023/Jan/26/padma-
• Recently, Prime Minister Narendra Modi
shri-awards-for-seven-from-andhra-
was conferred with the ‘Grand Cross of
2541463.html
the Legion of Honor’, by French President
21. “Global Security Initiative” is a security Emmanuel Macron. It is the highest
initiative under the aegis of French honor in military or civilian orders.
(1) USA (2) China • In June 2023, President Abdel Fattah El-
(3) Russia (4) India Sisi conferred Prime Minister Modi with

AMIGOS IAS 8
the highest state honor of Egypt, the • In April 2016, during his visit to Saudi
'Order of Nile.' Arabia, Prime Minister Narendra Modi
• Governor General of Papua New Guinea, was conferred Saudi Arabia's highest
Sir Bob Dadae conferred PM Modi with civilian honor- the King Abdulaziz Sash.
Grand Companion of the Order of Logohu The Prime Minister was conferred the
in May 2023. It is the highest honor in prestigious award by King Salman bin
Papua New Guinea. Abdulaziz.
• PM Narendra Modi has also been 24. Consider the indicators of the Global Multi-
conferred the highest honor of Fiji, Dimensional Poverty Index (MPI) given by the
Companion of the Order of Fiji in UNDP.
recognition of his global leadership. The I. Nutrition II. Child Mortality
award was conferred by PM Sitiveni III. Sanitation IV. Dirking Water
Rabuka of Fiji in May 2023. V. Years of Schooling VI. Electricity
• During his visit to Papua New Guinea in How many of the following indicators are part
2023, PM Modi was conferred with the of MPI?
Ebakl Award by President Surangel S. (1) Only three (2) Only Four
Whipps, Jr. of the Republic of Palau.
(3) Only Five (4) All Six
• Bhutan honored PM Modi with the
Answer: 4
highest civilian decoration, the Order of
Explanation:
the Druk Gyalpo in December 2021.
• According to the global Multidimensional
• Legion of Merit by the US Government,
Poverty Index (MPI) 2023, a total of 415
the award of the United States Armed
million people moved out of poverty in
Forces that is given for exceptionally
India within just 15 years from 2005/2006
meritorious conduct in the performance
to 2019/2021.
of outstanding services and achievements
was conferred on PM Modi in 2020. • It is a key international resource that
measures (annually) acute
• PM Modi received the prestigious King
multidimensional poverty across more
Hamad Order of the Renaissance in 2019.
than 100 developing countries.
The honor was conferred by Bahrain.
• First launched in 2010 by UNDP and
• Order of the Distinguished Rule of Nishan
Oxford Poverty and Human Development
Izzuddin- the highest honor of the
Initiative (OPHI), the global MPI advances
Maldives awarded to foreign dignitaries
SDG 1 - ending poverty in all its forms
was presented to PM Modi in 2019.
everywhere.
• Russia conferred Prime Minister Modi
• It also measures interconnected
with their highest civilian honor - the
deprivations across indicators related to
Order of St. Andrew Award in 2019.
SDGs 1, 2, 3, 4, 6, 7 and 11.
• In 2019, the Prime Minster was awarded
the Order of Zayed Award. This is the
highest civilian honor of the United Arab
Emirates.
• In the year 2018, when Prime Minister
Narendra Modi paid a historic visit to
Palestine, he was awarded the Grand
Collar of the State of Palestine Award. This
is the highest honor of Palestine awarded
to foreign dignitaries. 25. Which of the following statements is/are
• The same year, PM Modi was bestowed correct regarding the I2U2 grouping?
upon the State Order of Ghazi Amir I. It was started in 2021.
Amanullah Khan – the highest civilian II. Apart from India, the other members
honor of Afghanistan. include the USA, the UK, and Israel.

AMIGOS IAS 9
III. It is also known as the International Forum provide-students-e-courses-from-top-
for Economic Cooperation. universities-of-world-8897295/
Select the correct answer from the options 27. “Flamingo Festival,” often seen in the news,
given below. is celebrated at which of the following
(1) Only I and II (2) Only II and III locations?
(3) Only I and III (4) All I, II, III (1) Kolleru Lake
Answer: 3 (2) Pulicat Lake
Explanation: (3) Coringa Wildlife Sanctuary
• The I2U2 grouping was formed in October (4) Kambala Konda Wildlife Sanctury
2021. Hence, Statement I is correct. Answer: 2
• I2U2 is an alliance created by four Explanation:
countries namely – India, Israel, the • The ongoing Flamingo Festival (January 3–
United Arab Emirates (UAE) and the 5, 2023) is turning multiple locations in
United States (US). Hence, Statement II is Andhra Pradesh into some of the best
not correct. venues for wildlife viewing. Places like
• The four-nation coalition is also known as Pulicat Lake (Andhra Pradesh’s side),
the ‘International Forum for Economic Nelapattu Bird Sanctuary, Sullurpeta,
Cooperation’ or the ‘West Asian Quad’. Atikanithippa, and BV Palem are right now
Hence, Statement III is correct. teeming with visitors and birds.
26. Recently, with which one of the following e- • The annual Flamingo Festival celebrates
platforms, Andhra Pradesh Government has the arrival of thousands of birds who make
signed Memorandum of Understanding their annual migration to the Indian
(MoU) to provide students e-courses from subcontinent during the winter months.
top universities of world? This is the time when they breed, feed,
(1) Byju’s (2) edX and nest. The Indian shores and
(3) Coursera (4) LinkedIn Learning woodlands became their safe home for a
Answer: 2 few months.
Explanation: Source:
https://timesofindia.indiatimes.com/trav
• The Andhra Pradesh government and edX
el/travel-news/experience-the-best-of-
will jointly launch new technologies and
nature-at-the-ongoing-flamingo-festival-
pedagogies for teaching and learning. edX
in-andhra-
may work with Andhra Pradesh HED to
pradesh/articleshow/96714045.cms
explore training and learning needs,
identify learning gaps and curate MOOCs 28. Which of the following statements is correct
(minors and skill development courses) with respect to YSR Sampurna Poshan and
from within edX’s existing online campus YSR Sampurna Poshan Plus?
subscription catalogue with respect to the I. Both are implementing to achieve
requirement of the current curriculum of reduction in malnutrition and anemia in
both professional and conventional pregnant, lactating women and children of
degree programs. below 10 years.
• edX is a world leading e-learning platform II. YSR Sampoorna Poshana Plus+ Scheme is
that offers online courses from 170 + top being implemented in 77 Scheduled and
ranking institutions of the world, including Tribal Sub Plan Mandals of the state.
37 out of the top 50 universities, and top Select the correct answer from the options
four of the world namely MIT, Harvard, given below.
Oxford, and Cambridge. (1) Only I (2) Only II
Source: (3) Both I and II (4) Neither I Nor II
https://indianexpress.com/article/cities/ Answer: 2
hyderabad/ap-signs-mou-with-edx-to- Explanation:

AMIGOS IAS 10
• Chief Minister YS Jaganmohan Reddy • Monthly Ration Commodities provided by
launched YSR Sampurna Poshan and YSR YSR Sampurna Poshana:- 2 Kg Ragi Flour, 1
Sampurna Poshan Plus “Take-Home Kg Atukulu, 250 gm Jaggery, 250 gm
Ration distribution program” for pregnant Chikki, 250 gm Dry Dates, 3 Kg Rice, 1 Kg
and lactating women from his camp office Dal, Half Liter Cooking Oil, 25 Eggs, 5 Liters
in Tadepalli on August 2 and personally Milk.
handed over the goods to the • YSR Sampurna Poshan Plus Monthly
beneficiaries. Ration Commodities:- 1 Kg Ragi Flour, 2 Kg
• Government of Andhra Pradesh is Multi Grain Atta, 500 gm Jaggery, 500 gm
committed to ensure access to Chikki, 500 gm Dry Dates, 3 Kg Rice, 1 Kg
supplementary nutrition for all pregnant Dal, Half Liter Cooking Oil, 25 Eggs, 5 liters
women, lactating mothers, children aged of milk.
7-72 months through Anganwadi Centres Source: https://telugu.samayam.com/andhra-
and implemented in 55,607 Anganwadi pradesh/news/ap-cm-ys-jagan-launches-
Centres. To achieve reduction in ysr-sampoorna-poshana-take-home-
malnutrition and anaemia in children ration/articleshow/102368378.cms
(below 6 years) and pregnant, lactating 29. Which of the following universities hosted a
women, Government is implementing YSR three-day national conference on “Ancient
Sampoorna Poshana and YSR Sampoorna Telugu Literature: Historical, Social, and
Poshana Plus (+). Hence, Statement I is Cultural Perspectives” recently?
incorrect. (1) Andhra University
• The Poshanapoorna Poshana Plus+ (2) Acharya Nagarjuna University
Scheme is being implemented in 77
(3) Adikavi Nannaya University
Scheduled and Tribal Sub Plan Mandals,
(4) Yogi Vemana University
the YSR Sampoorna Poshana Scheme is
implemented in the remaining mandals of Answer: 3
the State. It is estimated 3.8 lakh Explanation:
beneficiaries in 77 tribal mandals and • Adikavi Nannaya University (AKNU)
26.36 lakh beneficiaries in the remaining hosted a three-day national conference
mandals in the State will be covered for on 'Ancient Telugu Literature: Historical,
providing ICDS services through 55,607 Social and Cultural Perspectives' at AKNU
Anganwadi Centers, in consultation with from February 2 to 4.
National Institute of Nutrition (NIN). • This conference was organised under the
Hence, Statement II is correct. joint auspices of AKNU Telugu
• Current nutrition programs designed to Department, Centre of Excellence in
ensure uniformity and greater coverage of Classical Telugu (Nellore), Central Institute
beneficiaries with recommended of Indian Languages (Mysore), Higher
nutritional standards, while providing Education Department of the Union
iron, protein, energy rich nutrition to Ministry of Education.
pregnant women, lactating mothers. • It invited research papers on the topic
Keeping in view of Under Weight or 'Ancient Telugu Literature: Historical,
Malnourished children and high Social and Cultural Perspectives' to be
prevalence of anaemia in pregnant and submitted to this National Conference
lactating mothers’ nutrition programs along with some other suggested topics.
have been designed . Source:
• To provide nutritious food to pregnant https://www.thehansindia.com/andhra-
and lactating women, the Andhra Pradesh pradesh/3-day-meet-on-ancient-telugu-
government spends Rs.850 per person literature-at-aknu-from-feb-2-
under YSR Sampurna Poshan, while it 776599?infinitescroll=1
spends Rs.1,150 per person for Sampurna
Poshan Plus.

AMIGOS IAS 11
30. Which one of the following commemorative • However, the Argentine general election
coin was released by President Draupadi in November 2023 led to a change in
Murmu featuring N.T. Rama Rao recently? president to Javier Milei, who had
(1) ₹50 (2) ₹75 committed to withdrawing the country's
(3) ₹100 (4) ₹125 membership application.
Answer: 3 33. Which of the following statements is/are not
Explanation: correct with reference to the “Five Eyes”
intelligence-sharing alliance?
 The release of a ₹100 commemorative
coin featuring N.T. Rama Rao (NTR) by I. The Five Eyes alliance includes the United
President of India Draupadi Murmu at the States, the United Kingdom, France,
RashtraPati Bhavan in New Delhi on Australia, and New Zealand.
Monday was a solemn occasion for the II. These partner countries share a broad
family members. range of intelligence with one another.
Source: Select the correct answer using the options
https://www.thehindu.com/news/nation given below:
al/andhra-pradesh/president-murmu- (1) Only I (2) Only II
releases-100-commemorative-coin-on-nt- (3) Both I and II (4) Neither I nor II
rama-rao-to-mark-his-birth- Answer: 2
centenary/article67245287.ece Explanation:
31. Which of the following group of countries are • The Five Eyes alliance comprises the
part of the “Golden Crescent”? United States, United Kingdom, Australia,
(1) Myanmar, Laos, and Thailand Canada, and New Zealand, sharing
(2) Saudi Arabia, UAE, and Qatar intelligence extensively.
(3) Afghanistan, Pakistan, and Iran • Canadian Prime Minister Justin Trudeau’s
(4) Argentina, Peru, Brazil allegations against the Indian government
Answer:3 regarding the assassination of a Sikh
Explanation: Canadian were based on information
shared within the ‘Five Eyes’ intelligence
 The “Golden Crescent” includes the
alliance.
countries of Afghanistan, Pakistan, and
• U.S. Ambassador David Cohen confirmed
Iran, which are known for their significant
the use of shared intelligence by Canada in
cultivation and production of opium
making these allegations.
poppy, a key raw material for heroin
production. • A Canadian official had previously
mentioned that the allegations were
32. With respect to expansion of the BRICS
supported by surveillance of Indian
membership, Which of the following
diplomats in Canada and intelligence from
countries is not part of BRICS at present?
a major ally, without specifying the ally.
(1) Argentina (2) Egypt
34. Consider the following countries.
(3) Ethiopia (4) Iran
I. Cook Islands II. Fiji
Answer: 1
III. Marshall Islands IV. Solomon Islands
Explanation:
V. Philippines
• In August 2023, at the 15th BRICS Summit,
How many of the above are part of the Forum
South African President Cyril Ramaphosa
for India-Pacific Islands Cooperation (FIPIC)?
announced that six emerging market
group countries (Argentina, Egypt, (1) All Five (2) Only Four
Ethiopia, Iran, Saudi Arabia, and the (3) Only Three (4) Only Two
United Arab Emirates) had been invited to Answer:2
join the bloc. Explanation:
• Full membership was scheduled to take India-Pacific Islands Cooperation (FIPIC):
effect on January 1, 2024.

AMIGOS IAS 12
• PM Modi co-chaired the third summit of • The General Data Protection Regulation
the Forum for India-Pacific Islands (GDPR) is an EU law on data protection
Cooperation (FIPIC) in Port Moresby, and privacy in the European Union (EU)
Papua New Guinea. and the European Economic Area (EEA).
• The Forum for India-Pacific Islands • It was approved by the EU in April 2016
Cooperation (FIPIC) was launched in and came into force on 25th May 2018.
November 2014 during the visit of Prime 36. Recently, Telangana and Andhra Pradesh
Minister Narendra Modi to Fiji. agreed to construct an iconic Hybrid Cable
• FIPIC was launched in line with India’s Act Bridge on the River Krishna at?
East Policy. (1) Jurala (2) Nagarjuna Sagar
• This initiative of India is also part of South- (3) Somasila (4) Srisailam
South Cooperation. Answer: 3
• The FIPIC involves India and 14 Pacific Explanation:
Island countries, namely, Fiji, Papua New • For the first time after the bifurcation of
Guinea, Tonga, Tuvalu, Kiribati, Samoa, Andhra Pradesh, the new state of
Vanuatu, Niue, the Federated States of Telangana and the residuary state of AP
Micronesia, the Republic of the Marshall agreed on one important project and
Islands, the Cook Islands, Palau, Nauru, conveyed their approval to the Union
and the Solomon Islands. Government.
• The key objective of the initiative is to • The issue on which both states have
expand India’s engagement in the Pacific shown great interest is the construction of
region. an iconic Hybrid Cable Bridge on the River
• India, through the FIPIC, has offered some Krishna at Somasila in Nagarkurnool
major assistance projects, which include district, bordering the two states.
setting up a special fund for adapting to • This bridge, when completed, would
climate change, setting up a trade office in reduce the travel time between
India, efforts to improve digital Hyderabad, Tirupati, and other major
connectivity, extending visas on arrival at towns in Andhra Pradesh.
Indian airports for all 14 Pacific Island
• Distance wise, it would be 80 km less. The
countries, cooperation in space
Centre has allocated Rs 500 crore in its
technology, and training the diplomats
2023-2024 budget.
from these countries, among others.
Source:
• Further, India has increased the annual
https://www.thehansindia.com/news/citi
“Grant-in-Aid” from USD 125,000 to
es/hyderabad/bridge-to-bring-telugu-
200,000 for each of these 14 Pacific Island
states-together-784288?infinitescroll=1
countries.
37. Consider the following statements with
• The first summit of FIPIC was held in Suva,
respect to Natural farming in Andhra
Fiji, in November 2014.
Pradesh.
• The second summit of FIPIC was held in
I. It is promoted by the Rythu Sadhikara
Jaipur in August 2015.
Samstha under the brand name of “AP
35. The General Data Protection Regulations, Community Manged Natural Farming.”
which have been in the news recently, are
II. AP got the first place in Jaivik India Awards
related to?
2023 organized by the International
(1) World Trade Organization Competence Centre for Organic
(2) European Union Agriculture (ICCOA).
(3) United States of America Which of the above statements is/are correct?
(4) India (1) Only I (2) Only II
Answer: 2 (3) Both I and II (4) Neither I Nor II
Explanation: Answer: 3

AMIGOS IAS 13
Explanation: (1) Only I (2) Only II
• For 2023, Andhra Pradesh has bagged (3) Both I and II (4) Neither I Nor II
three Jaivik India Awards in the national Answer: 3
level competitions organized by the Explanation:
International Competence Centre for • Garimella Balakrishna Prasad, an Indian
Organic Agriculture (ICCOA), Greater classical devotional singer and composer,
Noida. AP got first place in the category of received the Central Sangeet Natak
States promoting natural farming. It has Akademi Award 2023 from President
also won awards in two other categories. Draupadi Murmu.
More than 8 lakh farmers are practicing
• Prasad is the Asthana Vidwan of the
natural farming in the State. Hence,
Tirumala-Tirupati Devasthanams and
Statement I is correct.
Kanchi Kamakoti Peetham. He is the first
• A.P.C.N.F – primacy to farmers own one to be appointed in that capacity at
practices. In 2020, the name of the Ahobila Mutt.
programme was changed to A.P
• A disciple of Guru's Nedunuri Krishna
Community managed natural farming, to
Murthy, Prasad has been a pioneer in
emphasize farmers’ ownership. Even
composing and rendering Annamacharya
when the programme was called A.P Zero
Sankeerthanas. He is known for his
budget natural farming programme, our
innumerable contributions to the field of
farmers were following a mix of Z.B.N.F
music.
practices and other biological and non-
• He composed a record number of 1,000
chemical inputs : farmyard manure, vermi
Annamacharya Sankeerthanas till now,
compost, NADEP compost, dung from
out of which over 700 were recorded for
buffaloes, using inoculants like VAM, PSB,
TTD only, which is a record.
etc. It is to respect farmers’ choices in the
matter of practices. It is regarded all of Source:
them as NF farmers, as long as they are https://www.thehansindia.com/news/citi
not using synthetic chemical fertilisers, es/tirupathi/ttd-chairman-in-ayodhya-
pesticides, herbicides, weedicides. 852881?infinitescroll=1
• Andhra Pradesh Community Managed 39. Which of the following cities from Andhra
Natural Farming (APCNF) is being Pradesh got the 3rd rank in the Swachh Vayu
implemented by Rythu Sadhikara Sarvekshan 2023 Awards in the population
Samstha. Hence, Statement II is correct. category of 3–10 lakh?
Source1: (1) Visakhapatnam (2) Guntur
https://www.newindianexpress.com/stat (3) Anantapur (4) Vijayawada
es/andhra-pradesh/2023/aug/27/natural- Answer: 2
farming-andhra-pradesh-gets-3-jaivik- Explanation:
india-awards-2609163.html • Guntur city has been ranked 3rd in the
Source 2: https://apcnf.in/about-apcnf/ Swachh Vayu Sarvekshan 2023 in the
38. Which of the following statements is correct population category of 3–10 lakh.
With respect to Garimella Balakrishna • The Ministry of Environment, Forestry,
Prasad? and Climate Change (MoEF&CC) had
I. He received Sangeet Natak Akademi framed the guidelines for ranking cities
Award 2023. under the National Clean Air Programme
II. He is the Asthana Vidwan of the Tirumala- (NCAP) for the financial years 2022–23 to
Tirupati Devasthanams and Kanchi 2025–26.
Kamakoti Peetham. • Guntur Municipal Corporation
Select the correct answer from the codes Commissioner Keerthi Chekuri said, the
given below. city got the third rank because of its
continuous efforts to reduce pollution and
increase green cover.

AMIGOS IAS 14
• The civic body increased greenery from Development, and Inclusive
17% in 2021 to 30% in 2023. Earlier, the Multilateralism.
median on only 10 km of roads had plants. (4) BRICS Partnership for Global
It has now increased to 23 km. Avenue Sustainability, Shared Health and
plantation was spread over 20 km earlier, Innovative Technology.
which has now reached 30 km. Answer: 3
Source: Explanation:
https://www.thehindu.com/news/national/a  The theme of BRICS 2023 is: “BRICS and
ndhra-pradesh/guntur-city-ranks-third-in- Africa: Partnership for Mutually
swachh-vayu-sarvekshan-2023-thanks-to- Accelerated Growth, Sustainable
increased-green-cover-reduction-in-air- Development, and Inclusive
pollution/article67263855.ece Multilateralism”
40. The Andhra Pradesh Police has initiated 42. “Atlantic Declaration Action Plan (ADAPT)” is
'Operation Parivartan', a special operation focused on building a resilient, diversified,
undertaken to? and secure supply chain, reducing strategic
(1) Change the behaviour of the drunks and dependencies. This is a declaration between
drivers. (1) USA and European Union
(2) Crack down on cannabis cultivation in the (2) USA and France
state. (3) USA and UK
(3) Change the eve-teasing culture on the (4) USA and Germany
college premises.
Answer: 3
(4) Crack down on the illegal mining.
Explanation:
Answer: 2
• The United States and the United Kingdom
Explanation: have announced the Atlantic Declaration
• The Andhra Pradesh Police has initiated for a Twenty-First Century U.S.-UK
'Operation Parivartan', a special operation Economic Partnership.
undertaken to crack down on cannabis • The Atlantic Declaration is focused on
cultivation in the state. building a resilient, diversified, and secure
• The operation was led by the Andhra supply chain, reducing strategic
Pradesh Police, collaborating with the dependencies.
Special Enforcement Bureau (SEB) in • The partnership aims to leverage the
Visakhapatnam. energy transition and technological
• Operation Parivartan is aimed at curbing breakthroughs to drive shared growth,
the supply side of the ganja menace in the create employment opportunities, and
state. uplift communities.
Source: Atlantic Declaration Action Plan (ADAPT):
https://www.indiatoday.in/india/story/o • The ADAPT sets forth a comprehensive
peration-parivartan-andhra-pradesh- strategy to enhance economic growth,
police-ganja-cultivation-1911424-2022- competitiveness, and resilience while
02-10 prioritizing workers, businesses, climate,
41. What is the theme of the 15th BRICS summit, and national security.
which took place August 22–24, 2023, in • The plan encompasses five key pillars and
South Africa? involves regular high-level meetings to
(1) BRICS and Africa: Partnership for Global drive progress and increase ambition over
Security, Shared Destiny and Innovative time.
Ideas. 43. Consider the following statements with
(2) BRICS and Africa : Partnership for Global reference to International North South
Stability, and Innovative Growth. Transport Corridor (INSTC):
(3) BRICS and Africa: Partnership for Mutually
Accelerated Growth, Sustainable

AMIGOS IAS 15
I. Russia, India, and Iran are the founding • NATO was created by 12 countries from
member states of INSTC. Europe and North America on 4 April
II. It connects Russia with Iran and India via 1949.
the Black Sea. • Finland became NATO’s newest member
Which of the above statement(s) is/are today, upon depositing its instrument of
correct? accession to the North Atlantic Treaty
(1) Only I (2) Only II with the United States at NATO
(3) Both I and II (4) Neither I Nor II Headquarters in Brussels.
Answer: 1 • The US Senate India Caucus will introduce
legislation to give India ‘NATO plus five’
Explanation:
defence status.
• The International North– South Transport
• Currently, NATO Plus 5 is a security
Corridor (INSTC) is a 7,200-km long multi-
arrangement that brings together
mode network of ship, rail, and road
Australia, New Zealand, Japan, Israel, and
routes for moving freight between India,
South Korea.
Iran, Azerbaijan, Russia, Central Asia and
Europe. 45. Consider the following statements with
reference to ASEAN (Association of Southeast
• Russia, Iran & India signed the
Asian Nations):
International North-South Transport
Corridor (INSTC) project in 2002. Hence, I. ASEAN currently has 11 member
Statement I is correct. countries.
• It connects Russia with Iran and India via II. It primarily focuses on economic and
the Caspian Sea. Hence, Statement II is not military cooperation among its member
correct. states.
44. Which of these countries are part of the Which of the statements given above are
‘NATO plus five’ arrangement? correct?
(1) Australia, New Zealand, Japan, Israel and (1) Only I (2) Only II
South Korea (3) Both I and II (4) Neither I nor II
(2) India, New Zealand, Japan, Israel and Answer: 4
South Korea Explanation:
(3) India, New Zealand, Ukraine, Israel and • ASEAN was established in 1967 with the
South Korea signing of the ASEAN Declaration, but it
(4) Australia, New Zealand, Japan, Ukraine currently has 10 member countries.
and South Korea • It primarily focuses on economic and
Answer: 1 political cooperation, not military.
Explanation: 46. The Andhra Pradesh Guaranteed Pension
• The North Atlantic Treaty Organization Scheme (GPS) ensures government
(NATO) is a collective security system: its employees a monthly pension equivalent to
independent member states agree to how much of their last-drawn salary?
defend each other against attacks by third (1) 20 (2) 35
parties. During the Cold War, NATO (3) 50 (4) 60
operated as a check on the threat posed Answer: 3
by the Soviet Union. Explanation:
• At present, NATO has 31 member • Andhra Pradesh’s Guaranteed Pension
countries. These countries, called NATO System (GPS) blends elements from both
Allies, are sovereign states that come old and new pension schemes, offering
together through NATO to discuss political the advantages of a guaranteed pension
and security issues and make collective while not overly straining the state’s
decisions by consensus. finances.

AMIGOS IAS 16
• This innovative system holds the potential • The government of AP extended
to preserve India’s hard-won pension teleconsultation services to all PHCs and
reforms. YSR Village Health Clinics. Currently, 27
• A Hybrid Approach: The Andhra Pradesh telemedicine hubs are established in the
Guaranteed Pension System Bill, 2023, state.
recently approved by the state assembly, • 8,351 Village Health Clinics are the
introduces a unique blend of the Old spokespeople in the state.
Pension Scheme (OPS) and the New Source:
Pension Scheme (NPS) implemented in https://www.newindianexpress.com/stat
2004. es/andhra-pradesh/2023/Feb/08/andhra-
• Andhra’s Middle Path: Andhra Pradesh’s pradesh-ranks-1st-to-implement-e-
GPS offers a middle ground, preventing a sanjeevani-telemedicine-2545439.html
regressive return to the old scheme while 48. Who among the following received the
addressing concerns about NPS. Gadicherla Award in 2023?
• With 5.07 lakh government employees (1) Prof. K. Hemachandra Reddy
working in the state, the number of OPS (2) Mandali Buddha Prasad
employees is more than 2.02 lakh, while (3) Prof. G. Gnana Mani
that of CPS employees is more than 3.73
(4) Raavi Sarada
lakh. The bill entailed a financial
Answer: 2
implication of approximately Rs 2,500
crore by 2040. Explanation:
• The GPS subscriber will get a monthly • Former Deputy Speaker Mandali Buddha
guaranteed pension at the rate of 50% of Prasad was honoured with the Gadicherla
the last drawn basic pay. Award in 2023. Governor S. Abdul Nazeer
presented the award to him at a function
• It will also ensure a monthly spouse
held at Krishna University (KRU) in
pension at the rate of 60% of the
Rudravaram village in Krishna district.
guaranteed pension, in case of a shortfall
in the annuity received by the spouse of • The programme was jointly organised by
the deceased APGPS subscriber. the Gadicherla Foundation and the
Andhra Pradesh Library Association,
Source:
commemorating the 140th birth
https://www.newindianexpress.com/states/a
anniversary of freedom fighter Gadicherla
ndhra-pradesh/2023/sep/28/guaranteed-
Harisarvottama Rao, said Chandrasekhar
pension-system-bill-passed-by-ap-
Kalkura, president of the foundation.
assembly-2618924.html
• Harisarvottama Rao was called the ‘Father
47. Andhra Pradesh was recognised for its best
of the Library Movement’ as he devoted a
efforts in the implementation of e-
part of his life to the library movement. He
Sanjeevani Telemedicine services in India.
encouraged the youth to cultivate the
What is the rank of Andhra Pradesh in its
habit of reading books to gain knowledge.
implementation?
Source:
(1) 1st (2) 2nd
https://www.thehindu.com/news/national/a
(3) 3rd (4) 4th
ndhra-pradesh/gadicherla-award-
Answer: 1 presented-to-mandali-buddha-
Explanation: prasad/article67307855.ece
• Andhra Pradesh stood in first place in the 49. The Crop Cultivators Rights Act of Andhra
implementation of e-Sanjeevani Pradesh 2019 tries to mutually benefit both
Telemedicine services in India, the landowner and cultivator. The Crop
contributing 32% of teleconsultations, i.e., Cultivators Rights Card be issued for a period
3.1 crores, against 9.7 crores of total of
teleconsultations across India. (1) 15 Months (2) 11 Months
(3) 18 Months (4) 24 Months

AMIGOS IAS 17
Answer: 2 III. Sudan IV. Israel
Explanation: Select the correct answer from the options
given below.
(1) Only I and II (2) Only II and III
(3) Only I, II, and III (4) All the above
Answer: 3
Explanation:

Source:
https://www.newindianexpress.com/stat
es/andhra-
pradesh/2023/May/10/amend-crop-
cultivator-rights-act-to-protect-tenant-
farmers-rsv-to-andhra-pradesh-govt-
2573970.html
50. Which of the following government
departments in Andhra Pradesh has signed a
Memorandum of Understanding (MoU) with
the International Baccalaureate recently?
(1) Department of Industries
(2) Department of Education
(3) Department of Skill Development
(4) Department of Panchayat Raj
Answer: 2  The Red Sea is bordered by multiple
Explanation: countries including Saudi Arabia, Egypt,
• Andhra Pradesh Wednesday set the road Sudan, Yemen, Djibouti, and Eritrea.
for imparting quality education of global 52. Consider the following statements with
standards to government school students respect to Shanghai Cooperation
through the International Baccalaureate Organization (SCO):
(IB) syllabus that nurtures critical thinking, I. The SCO was founded in 2001 by China,
creativity, and global citizenship. Kazakhstan, Kyrgyzstan, Russia, Tajikistan,
• The state government has signed an and Uzbekistan.
agreement with IB Organisation in this II. India and Pakistan became full members
regard in a videoconference meeting held of the SCO in 2017.
at Chief Minister Y S Jagan Mohan Reddy’s III. The SCO’s main focus is on security
office. cooperation, including combating
• Education Department officials said that terrorism and extremism.
with this partnership, the IB syllabus will Which of the statements given above are
be made available to students in over correct?
45,000 government schools. (1) Only I, II (2) Only II, III
Source: (3) Only I, III (4) All the above
https://indianexpress.com/article/cities/hyde Answer: 4
rabad/andhra-pradesh-international-
Explanation:
baccalaureate-syllabus-govt-schools-
• The SCO was established by six founding
8948823/
members in 2001, and India and Pakistan
51. Which of the following are border countries
joined as full members in 2017.
of the Red Sea?
I. Saudi Arabia II. Eritrea

AMIGOS IAS 18
• The organization’s primary focus is on Answer: 3
security cooperation, particularly in Explanation:
countering terrorism and extremism. What is the new Global Financing Pact?
53. Consider the following statements with • The aim of the Summit is to lay the
respect to Nuclear Suppliers Group (NSG): groundwork for a new financial system
I. The NSG was formed in the wake of the suited to the common challenges of the
nuclear tests conducted by India in 1974. 21st century, such as fighting inequalities
II. India subsequently joined NSG in 1998. and climate change and protecting
III. There are 48 participating governments at biodiversity. The Summit is co-organized
present. by France and India.
Which of the above given statements is/are • In other words, the aim of this Summit is
correct? to concurrently tackle the challenges of
(1) Only I and II (2) Only II and III climate change, biodiversity protection,
(3) Only I and III (4) All I, II, III and the fight against inequality in order to
achieve the Sustainable Development
Answer: 3
Goals (SDGs).
Explanation:
Four major objectives for this Summit have
• The Nuclear Suppliers Group (NSG) is a
been announced and will be followed up by
transnational body comprised of nuclear
four working groups:
supplier countries that aim to control the
• Restoring fiscal space to countries facing
proliferation of nuclear weapons by
short-term difficulties, especially the most
curbing the export of nuclear weapons
indebted countries
development materials and related
technology. • Promoting private sector development in
low-income countries
• It seeks to improve the existing safeguards
on existing nuclear materials. • Encouraging investment in green
infrastructure for the energy transition in
• The NSG was formed in the wake of the
emerging and developing countries
nuclear tests conducted by India in May
1974 which proved that certain non- • Mobilising innovative financing for
weapons nuclear technology could be countries vulnerable to climate change.
used to develop nuclear weapons. Hence, 55. ‘Climate Shift Index’ (CSI) model, recently
Statement I is correct. seen in news has been developed by:
• The group had its first meeting in (1) Sustainable Development Solutions
November 1975. A series of meetings held Network (SDSN)
in London produced agreements on (2) United Nations Framework Convention on
export guidelines. Climate Change (UNFCCC)
• Although initially there were only 7 (3) United Nations Environment Programme
countries as members, there are 48 (UNEP)
participating governments as of now. (4) Climate Central
Hence, Statement III is correct. Answer: 4
• India is not one of them. Hence, Explanation:
Statement II is not correct. • The Climate Shift Index model has been
54. The new Global Financing Pact aims to lay the developed by a U.S Non-profit
groundwork for a new financial system suited organization called Climate Central.
to the common challenges of the 21st • It reveals how much climate change
century, such as fighting inequalities and influences the temperature on a particular
climate change and protecting biodiversity. day.
The Summit is co-organized by? 56. Supporting Andhra’s Learning
(1) USA and India (2) Japan and India Transformation (SALT) is a project
(3) France and India (4) Egypt and India

AMIGOS IAS 19
undertaken by the Government of Andhra
Pradesh with support from
(1) Japan International Cooperation Agency
(JICA).
(2) World Bank.
(3) Asian Infrastructure and Investment Bank.
(4) New Development Bank.
Answer: 2
Explanation:
• Recently, the World Bank extended an
unconditional loan of $250 million for the
SALT project in Andhra Pradesh.
• Supporting Andhra’s Learning
Transformation (SALT) is a project
undertaken by the Government of Andhra
Pradesh with support from the World
Bank.
• The SALT is the first project in the school
education sector to be funded by the
World Bank without any precondition.
• To transform the state’s school education
system by strengthening the quality of
foundational learning through various
pathways including improving teacher
professional development, classroom-
based assessments, and early childhood
education.
• Establishing and strengthening foundation Source:
schools is in tune with the National https://www.thehindu.com/news/nation
Education Policy, 2020. al/andhra-pradesh/mecon-ltd-to-select-
Source: construction-firm-through-global-
https://www.thehindu.com/news/nation tenders-for-central-tribal-university-of-
al/andhra-pradesh/education- ap/article67493342.ece
department-officials-upbeat-over-world- 58. Consider the following government
bank-validation-of-schemes-introduced- beneficiaries:
under-salt/article66673010.ece I. Tailors of all communities
57. Central Tribal University is coming up in II. Rajakas (washermen)
Andhra Pradesh in III. Nayee Brahmins
(1) Vizianagaram District IV. Kapus
(2) Parvatipuram Manyam District Who among the above is eligible for the
(3) Alluri Seetharamaraju District Jagananna Chedodu Scheme?
(4) Visakhapatnam District (1) Only I, II and III (2) Only II and III
Answer: 1 (3) Only III and IV (4) All the above
Explanation: Answer: 1
Explanation:
• The Jagananna Chedodu Scheme is a state
government welfare scheme launched on
June 10, 2020, for small businessmen
residing in Andhra Pradesh (AP).

AMIGOS IAS 20
• This scheme is implemented by the B.C. • (i) supervision of the functioning of the
Welfare Department, Govt. of Andhra Godavari River Management Board and
Pradesh. Krishna River Management Board.
• The beneficiaries of the scheme include • (ii) planning and approval of proposals for
Tailors (all communities), Rajakas construction of new projects, if any, based
(washermen), and Nayee Brahmins on Godavari or Krishna River water, after
(barbers) of the state. getting the proposal appraised and
• The main objective of this scheme is to recommended by the River Management
provide financial assistance to small shop Boards and by the Central Water
owners whose businesses were affected Commission, wherever required.
by the COVID pandemic. • (iii) resolution of any dispute amicably
• The beneficiary will be provided a one- arising out of the sharing of river waters
time amount of ₹ 10,000 per year for a through negotiations and mutual
period of 5 years, i.e., ₹ 50,000/- in five agreement between the successors
instalments. States.
• This fund can be utilized by the • (iv) reference of any disputes not covered
beneficiaries for purchasing tools, under Krishna Water Disputes Tribunal, to
equipment, and other essentials to grow a Tribunal to be constituted under the
their source of income and work Inter-State River Water Disputes Act, 1956
establishment. Source:
Source: https://www.newindianexpress.com/hyd
https://telugu.hindustantimes.com/andh erabad/2023/Dec/03/nsp-row-centre-to-
ra-pradesh/kurnool-cm-jagan-tour- hold-meeting-on-december-6-
jagananna-chedodu-funds-released-on- 2638275.html
october-19th-121697548014982.html 60. The Fishbone model of irrigation system has
59. Who is not a member of the Apex Council for been in the news with respect to the Krishna
the supervision of the functioning of the Wildlife Sanctuary (KWS). It is known for
Godavari River Management Board and restoration of?
Krishna River Management Board? (1) Rare fish found in this sanctuary
(1) Minister of water resource, Government (2) Mangrove cover in this area
of India (3) Crops that can sustain marine waters
(2) Chief Minister of State of Andhra Pradesh (4) Pythons and other kind of snakes
(3) Chief Minister of State of Telangana Answer: 2
(4) Governors of both states Explanation:
Answer: 4 • The Fishbone model of irrigation has
Explanation: helped restore the mangrove cover in the
 The Central Government shall constitute Krishna Wildlife Sanctuary (KWS) on
an Apex Council for the supervision of the nearly 6,000 hectares since 2006.
functioning of the Godavari River • The method is used to artificially inundate
Management Board and Krishna River areas that do not get regular tidal
Management Board. inundation. By flooding dried-up wetlands
• The Apex Council shall consist of (a) near the intertidal zones, new mangroves
Minister of Water Resources, Government can be reforested.
of India—Chairperson; (b) Chief Minister • In this model, the water from a creek is
of State of Andhra Pradesh—Member; (c) diverted to the targeted site through
Chief Minister of State of Telangana— feeder and field channels, converting the
Member. barren land with high saline content into
• The functions of the Apex Council shall fertile land to support the survival of
include mangrove plant species.

AMIGOS IAS 21
• The shape of the fishbone allows the development among West African
water to reach every corner of the field nations.
channels. The seeds of the mangrove • The Secretariat, headquartered in Abuja,
plant species are sown on the banks of all Nigeria.
the channels once the salinity is reduced • ECOWAS consists of 15 member states:
to the required level for the survival of the Benin, Burkina Faso, Cape Verde, Cote d’
plants. Ivoire, The Gambia, Ghana, Guinea,
Guinea Bissau, Liberia, Mali, Niger,
Nigeria, Sierra Leone, Senegal, and Togo.
Primary Goals and Objectives:
• Promoting economic integration among
member states.
• Facilitating the free movement of people,
goods, and services.
• Enhancing regional cooperation in various
sectors, including industry, transport,
Source: energy, and finance.
https://www.thehansindia.com/andhra- • Fostering a borderless region governed by
pradesh/amalapuram-fishbone-model- democratic principles, the rule of law, and
channels-for-mangrove-expansion- good governance.
800487 • Addressing security and political
61. Which of the following is/are correct challenges through collaboration and
statement about the Economic Community of conflict resolution.
West African States (ECOWAS)? 62. Which of the following pairs is correct with
I. ECOWAS has 15 member countries. regard to the missiles and their respective
II. It was established in 1975 with the signing countries of origin??
of the Treaty of Lagos. (1) Tomahawk: China
Select the correct answer using the options (2) SCALP: India
given below. (3) S-400: United States
(1) Only I (2) Only II (4) Hwasong 18 : North Korea
(3) Both I and II (4) Neither I nor II Answer: 4
Answer: 3 Explanation:
Explanation: • Tomahawk missile is developed and
• Recently, the Economic Community of produced by the United States.
West African States (ECOWAS), has • The SCALP-EG/Storm Shadow missile is a
emerged at the forefront of international joint development of France and the
attention due to its active involvement in United Kingdom.
addressing the aftermath of the coup in • The S-400 missile is developed and
Niger. produced by Russia.
• This coup, which ousted the • Hwasong Missiles are series of missiles
democratically elected President and developed by North-Korea.
ushered in a military coup, has raised 63. Consider the following islands.
alarm bells globally.
I. Scarborough Shoal
• The ECOWAS, also known as CEDEAO in
II. Paracel Islands
French, is a regional intergovernmental
organization established in 1975. III. Spratly Islands
• Created through the Lagos Treaty, IV. Natuna Islands
ECOWAS aims to foster economic Which of the above islands are located in
integration, cooperation, and South-China Sea?
(1) Only I, II, III (2) Only II, III, IV

AMIGOS IAS 22
(3) Only I, II, III (4) All the above • The new Russian submarine-launched
Answer: 4 Intercontinental Ballistic Missile (ICBM) is
Explanation: known as the Bulava.
 All the given islands are located in South- • The Bulava missile is designed to be
China Sea. The Islands in the South China deployed on Russia's Borey-class ballistic
Sea from North to South: Paracel Islands – missile submarines (SSBNs).
Scarborough Shoal – Spratly Islands – • Features: It measures 12–13.5 metres in
Natuna Islands. length, has a diameter of about 2.0
meters, and has a launch weight of 36,800
kilograms.
• The Bulava is a three-stage solid-fuel
missile capable of carrying multiple
warheads, each intended to strike a
different target.
• It can carry up to six Multiple Independent
Re-entry Vehicles (MIRVs), which have the
ability to maneuver in-flight and retarget
to evade hostile air defenses. The missile
is reported to have an accuracy of 350
64. "Wushu," a martial art form, belongs to meters.
which country?? 66. Consider the following statements with
(1) Japan (2) China respect to Jagananna Arogya Surakhsa
(3) Vietnam (4) Korea campaigns:
Answer: 2 I. It happened in 5-phases at Grama Ward
Explanation: Sachivalayam level.
• India recently won 10 gold medals at II. Diagnostic and specialist services have
Moscow Star Wushu Championship 2023. also been offered under this campaign.
• Wushu is a collection of Chinese martial Which of the above statements is/are correct?
arts. (1) Only I (2) Only II
• Wushu in Chinese literally means “martial (3) Both I and II (4) Neither I Nor II
technique”. Answer: 3
• It has a history dating back thousands of Explanation:
years to the Qin Dynasty.
• The International Wushu Federation was
formed in 1990 to govern the distinct
types of martial arts under the “wushu”
banner.
65. Which intercontinental ballistic missile
capable of carrying nuclear warheads was
recently test-launched by Russia from a
submarine?
(1) Topol (2) Yars
(3) Bulava (4) Sarmat
Answer: 3
Explanation:
• Russia recently carried out a successful
test launch of the Bulava ballistic missile,
designed to carry nuclear warheads.
Bulava ballistic missile:

AMIGOS IAS 23
68. Consider the following statements regarding
YSR Sunna Vaddi Scheme.
I. It was launched in the year 2020.
II. The applicant must be a Self-Help Group
Member.
III. The applicant must belong to the below
poverty line.
Which of the above given statements is/are
correct?
(1) I only (2) I and II only
(3) II and III only (4) All the above
Answer: 4
Explanation:
• Prime Minister Narendra Modi interacted
with a member of a women Self Help
Group from Andhra Pradesh About YSR
Sunna Vaddi Scheme:
• This scheme was launched in the year
2020.
• The scheme intends to provide zero
67. Which of the following Railway Station in
interest loans for women in self-help
Andhra Pradesh has been awarded IGBC’s
groups.
‘Green Railway Station’ certification with
• To reduce interest burden on poor SHG
platinum rating?
women, it is proposed to pay interest
(1) Visakhapatnam (2) Tenali
portion for the FY 2019-20 on SHG bank
(3) Vijayawada (4) Tirupati loan amount outstanding under YSR
Answer: 3 Sunna Vaddi.
Explanation: • The main objective of the scheme is to
• Vijayawada Railway Station was awarded provide a free credit loan to all of the self-
the ‘Green Railway Station’ certification help groups of the Andhra Pradesh state.
with the highest rating of Platinum by the • All SHG women from both the rural and
Indian Green Building Council (IGBC) for its urban areas having the bank loan accounts
efforts in improving environmental will receive an outstanding amount up to
standards and providing eco-friendly Rs.5.00 lakhs under the YSR Sunna Vaddi
services to passengers. Pathakam.
• This is an upgrade of the station’s ratings, • SHG loan accounts declared NPA as on
from gold in 2019 to platinum in 2023. 31/03/2019 as per SERP and MEPMA data
• Divisional Railway Manager Narendra A. base will not get the YSR Sunna Vaddi
Patil said this is second in the South- Pathakam.
Central Railway (SCR) zone after Eligibility of the scheme:
Secunderabad, to have bagged the • Only the permanent resident of Andhra
platinum rating. Pradesh state can avail for the scheme
Source: benefits.
https://www.thehindu.com/news/national/a • The applicant must be a Self-Help Group
ndhra-pradesh/vijayawada-gets-igbcs- Member.
green-railway-station-certification-with- • The applicant must belong to the below
platinum-rating/article67273356.ece poverty line that is only the Poor SHG
member will get the chance to apply for
the scheme.

AMIGOS IAS 24
• SHG women (both rural and urban) with • On the occasion of the observed World
bank loan accounts outstanding amount Day Against Child Labor, the Andhra
up to Rs.5.00 lakhs are eligible. Pradesh Crime Investigation Department
Source: (CID) unveiled a poster against child
https://www.thehindu.com/news/nation labour.
al/andhra-pradesh/govt-to-release-1353- • The AP CID is observing June as the action
crore-under-sunna-vaddi-scheme- month against child labour and
today/article67181302.ece conducting Operation Svechha Phase-II
69. Consider the following universities or from June 1 to June 30 in coordination
institutions: with local police and all stakeholder
I. KL University departments across the state.
II. Andhra University • Speaking on the occasion, CID chief and
III. Venkateswara University additional director general of police N
Sanjay said that this operation is a
IV. Vignans Foundation for Science,
continuation of Operation Svechaa phase
Technology and Research
1. “During phase-I, the CID made
Which of the above institutions have bagged a
significant efforts to rescue children,
place in the overall top 100 institutions in the
which received appreciation from the
country in the National Institutions Ranking
government,” he added.
Framework 2023?
Source:
(1) Only I and II (2) Only I and III
https://www.newindianexpress.com/vija
(3) Only I and IV (4) Only II and III yawada/2023/Jun/13/operation-svechha-
Answer: 1 phase-2-as-many-as-326-children-
Explanation: including-37-girls-rescued-2584538.html
• Only two institutions from Andhra 71. Which of the pairs of “Places in News:
Pradesh have bagged a place in the overall Country” are correct??
top 100 institutions in the country in the I. Bakhmut: Russia
National Institutions Ranking Framework II. Gaziantep: Turkey
2023.
III. Nusantara: Indonesia
Top 100 institutions category:
Select the correct answer from the options
• KL University-50 given below.
• Andhra University- 76 (1) Only I and II (2) Only II and III
Top 100 universities category: (3) Only III and I (4) All I, II, III
• KL University-28 Answer: 2
• Andhra University-43 Explanation:
• Venkateswara University-60 • Bakhmut is a city in eastern Ukraine.
• Vignans Foundation for Science, • It is an administrative centre and an
Technology and Research-75 industrial hub in the Bakhmut Raion of
70. “Operation Svechha Phase-II” has been Donetsk region. Hence, Pair I is not
launched in Andhra Pradesh recently, which correct.
is aimed? • Gaziantep, which is still informally called
(1) Freeing up the marine ecosystem by Antep, is a major city in south-central
decreasing the usage of nets. Turkey. It is the capital of the Gaziantep
(2) Restricting human activities in wildlife Province and the sixth-most populous city
sanctuaries to provide space for wildlife. in Turkey. Hence, Pair II is correct.
(3) Against child labour. • Nusantara is the future capital of
(4) Aimed against domestic violence. Indonesia scheduled to be inaugurated in
Answer: 3 2024. Nusantara is located on the east
Explanation: coast of Borneo, which is the world’s

AMIGOS IAS 25
third-largest island. Hence, Pair III is • The territory is internationally recognised
correct. as part of oil-rich Azerbaijan, but its
72. Which two bodies of water does the inhabitants are mostly ethnic Armenians
Tsushima Strait connect? with their own government that has close
(1) East China Sea and the Philippine Sea ties to the government in neighbouring
(2) South China Sea and the Sea of Japan Armenia but is not officially recognised by
it or other U.N. member states.
(3) East China Sea and the Sea of Japan
• Armenian Christians claim a lengthy
(4) South China Sea and the Philippine Sea
history of supremacy in the region that
Answer: 3
dates back several centuries before Christ.
Explanation:
• Azerbaijan, whose people are primarily
 The Tsushima Strait connects the East Muslim, also connects its historical
China Sea and the Sea of Japan in the identity to its territory. It accuses
northwest Pacific Ocean. Armenians of forcibly displacing Azeris
who resided nearby in the 1990s. It wants
full control of the enclave and has
suggested that ethnic Armenians obtain
Azeri passports or leave.
74. “The Wagner Group” often seen in news is
(1) A China-backed hacking group
(2) A Russian paramilitary organisation
(3) A U.S based non-governmental militia
organisation
(4) Iranian hacker group
Answer: 2
Explanation:
73. “Nagorno-Karabakh issue” is often  The Wagner Group also known as PMC
mentioned in news related to? Wagner is a Russian paramilitary
(1) Ukraine and Russia organisation. It is believed to have been
(2) Azerbaijan and Armenia founded in 2014.
(3) China and India 75. The “East Container Terminal (ECT) project”
(4) Israel and Syria recently in the news, is located in which of
Answer: 2 the following country?
Explanation: (1) India (2) Iran
• Azerbaijan announced the establishment (3) Djibouti (4) Sri Lanka
of a roadblock at the opening of the Lachin Answer: 4
corridor, the sole land route connecting Explanation:
Armenia and Nagorno-Karabakh, which • After over two years since Sri Lanka’s
was followed by reports of border gunfire decision to cancel a joint memorandum of
by both Azeri and Armenian forces. understanding (MoU) between India and
What is Nagorno-Karabakh? Japan for the East Container Terminal
• Nagorno-Karabakh, often known as (ECT) project in Colombo, the three
Artsakh by Armenians, is a hilly region in nations are now exploring avenues to
the South Caucasus. rejuvenate trilateral collaboration.
• After the fall of the Russian Empire in • The cancellation of the MoU, estimated at
1917, it was claimed by both Azerbaijan $500 million, and the suspension of the
and Armenia, and it has remained a Japanese-funded light rail transit (LRT)
source of contention ever since. project strained relations between Tokyo
and Colombo.

AMIGOS IAS 26
76. Adani Group began construction on the taking legal action, and also undertaking
nation's first integrated data centre and sensitisation programmes.
technology business park in? Source:
(1) Visakhapatnam (2) Tirupati https://www.deccanchronicle.com/natio
(3) Vijayawada (4) Guntur n/in-other-news/180523/ap-cid-
Answer: 1 launches-swabhiman-transgender-
Explanation: persons-protection-helpline.html
• In Visakhapatnam, the Adani Group began 78. Gundlakamma Project is located in which of
construction on the nation's first the following districts??
integrated data centre and technology (1) Prakasam (2) Nandyala
business park. (3) Bapatla (4) Palnadu
• The park would be the first of three Answer: 1
technological zones planned in Vizag. Explanation:
• It houses a skill development centre and • Power failure halted efforts to stop the
has a potential for 200+ MW of data water flow from the damaged second gate
centres, fueled by up to 100% renewable of the Gundlakamma project in Prakasam
energy. district.
Source: About Gundlakamma Project:
https://cio.economictimes.indiatimes.co • This project is also known as Kandula
m/news/big-data/adani-starts-work-on- Obula Reddy Gundlakamma Reservoir
data-centre-at-vizag-in-andhra- Project
pradesh/100001292. • It is formed across Gundlakamma river
77. Andhra Pradesh Government’s “Helpline near Chinnamallavaram Village of
Number 1091” is associated with? Maddipadu Mandal in Prakasam District.
(1) Anti-Ragging • This Project is envisaged to utilize 12.845
(2) Anti-corruption TMC of water to provide irrigation
(3) Public Grievances facilities to 62,368 Acres in Kharif and
(4) Swabhiman Transgender Persons 80,060 Acres in Rabi seasons in 6 Mandals
Protection cell of Prakasam District.
Answer: 4 Source:
Explanation: https://www.deccanchronicle.com/natio
n/current-affairs/091223/floodwaters-
• Andhra Pradesh CID launched Swabhiman
damage-gundlakamma-projects-second-
Transgender Persons Protection Helpline
gate-in-prakasam-distr.html
number 1091 in the CID office at Guntur
on Wednesday. AP CID additional DGP N. 79. The Andhra Pradesh government has
Sanjay proposed the helpline, which released approximately 10 million fish into
aimed to protect the safety and dignity of the state’s water bodies to combat mosquito-
transgender persons by ending violence, borne diseases like malaria and dengue.
preventing discrimination against them, Which of the following fish species is known
including abuse and physical violence, as a biological agent for controlling mosquito
protecting their rights, monitoring cases larvae?
of crimes against them, and taking legal (1) Salmon (2) Gambusia
action. (3) Catfish (4) Catla Fish
• This helpline would work in accordance Answer: 2
with the Transgender Persons (Protection Explanation:
Rules) Act-2019 and the Transgender • The Andhra Pradesh government has
Persons (Protection Rules)-2020 through released approximately 10 million
which they would analyse the complaints Gambusia fish into the state’s water
of transgender persons, counselling, bodies to combat mosquito-borne

AMIGOS IAS 27
diseases like malaria and dengue. The fish, Source:
also known as mosquitofish, is widely used https://timesofindia.indiatimes.com/city/
as a biological agent for controlling vijayawada/ap-to-reap-benefits-of-3-
mosquito larvae. industrial-
• However, the release of these invasive corridors/articleshow/102817993.cms
alien fish species has raised concerns 81. Which of the following statements is correct
about the potential harm that will be with respect to Nuclear Test Ban Treaty?
sustained by native species that abound in I. It banned Nuclear Weapons Tests in the
the state’s freshwater bodies. Atmosphere, in Outer Space,
• Andhra Pradesh reported approximately Underground and Under Water.
6,391 dengue cases and 2,022 malaria II. India, Pakistan, and North Korea are not
cases in 2022, according to the Union signatories to this treaty.
Ministry of Health and Family Welfare, Select the correct answer from the options
Government of India (MoHFW). given below.
• However, the cases have risen (1) Only I (2) Only II
significantly, according to local news (3) Both I and II (4) Neither I Nor II
reports: there have been 2,339 dengue
Answer: 2
and 1,630 malaria cases registered in the
Explanation:
state in the last six months.
Nuclear Test Ban Treaty:
Source:
https://www.downtoearth.org.in/news/ • Russian lawmakers will consider revoking
wildlife-biodiversity/gambusia-this- the ratification of a global nuclear test
solution-could-actually-be-an-invasive- ban.
problem-90665 • Russia could move to resume nuclear tests
80. Which of the following industrial nodes are to try to discourage the West from
present in Andhra Pradesh along the continuing to offer military support to
Chennai-Bangalore Industrial Corridor? Ukraine.
(1) Kopparthy Node About the Nuclear Test Ban Treaty:
(2) Donakonda Node • Signed in Moscow in August 1963.
(3) Orvakallu Node • Initial Parties (had Veto power): USA, UK,
USSR. (later 100 more nations joined)
(4) Krishnapatnam Node
• Formally called Treaty Banning Nuclear
Answer: 1
Weapons Tests in the Atmosphere, in
Explanation:
Outer Space, and Under Water. (Didn’t
ban underground testing). Hence,
Statement I is incorrect.
• Trigger: Cuban Missile Crisis.
• 1977: Negotiations started on the
Comprehensive Nuclear Test Ban Treaty
• Draft CTBT passed in UNGA in 1996.
• To enter into force, it needs to be signed
by all nuclear powers and 44 members of
the Conference on Disarmament who
have nuclear reactors.
• By 2007 all of these countries but three
(India, Pakistan, and North Korea) had
signed, though 10 of those that had signed
had not ratified, including the United
States and China. Hence, Statement II is
correct.

AMIGOS IAS 28
82. The “C+C5 grouping”, is a China plus 84. With reference to the Indian Ocean Rim
(1) Five Central American Republics Association (IORA), consider the following
(2) Five Central Asian Republics statements:
(3) Five ASEAN Countries I. IORA is an organization of 23 coastal
(4) Five European Countries countries bordering the Indian Ocean.
Answer: 2 II. ‘Reinforcing Indian Ocean Identity’ was
the banner theme at the Indian Ocean Rim
Explanation:
Association’s (IORA) Council of Ministers
• The C+C5 grouping, which includes China
(COM) held in Colombo on October 11,
and central Asian countries (Kazakhstan,
2023.
Kyrgyzstan, Tajikistan, Turkmenistan, and
Which of the statements given above are
Uzbekistan), focuses on fostering
correct?
economic cooperation and regional
integration. (1) Only I (2) Only II
• These countries are strategically located (3) Both I and II (4) Neither I Nor II
along the ancient Silk Road, and their Answer: 3
partnership with China aims to enhance Explanation:
trade, investment, and infrastructure • Context: ‘Reinforcing Indian Ocean
development in the region. Identity’ was the banner theme at the
• This collaboration promotes economic Indian Ocean Rim Association’s (IORA)
growth and facilitates the flow of goods, Council of Ministers (COM) held in
services, and capital. Colombo on October 11, that was
• The C+C5 grouping is closely tied to attended by foreign ministers and senior
China’s ambitious Belt and Road Initiative. officials of the 23-nation grouping of
Central Asian countries serve as important countries.
nodes within the BRI, as they offer The IORA:
connectivity between China, Europe, and • The Indian Ocean Rim Association includes
the Middle East. 23 countries from Africa, West Asia, South
• Energy cooperation is a significant aspect Asia, Southeast Asia, Australia and littoral
of the C+C5 grouping. Central Asian states situated in and around the Indian
countries possess substantial energy Ocean.
reserves, particularly in the form of oil, • IORA’s membership includes 23 countries:
natural gas, and uranium. Australia, Bangladesh, the Comoros,
• China, as a rapidly growing economy, France, India, Indonesia, Iran, Kenya,
requires a stable energy supply, and these Madagascar, Malaysia, the Maldives,
countries offer a potential source. Mauritius, Mozambique, Oman,
83. Which country is developing a smart city at Seychelles, Singapore, Somalia, South
“Gelephu” on the border with India? Africa, Sri Lanka, Tanzania, Thailand, the
(1) Nepal (2) Bangladesh UAE and Yemen.
(3) China (4) Bhutan • It also has 11 dialogue partners: China,
Egypt, Saudi Arabia, Germany, Italy, Japan,
Answer: 4
South Korea, Russia, Türkiye, the U.K. and
Explanation:
the U.S.
• Recently, Bhutan’s King has announced
• The grouping, whose apex body is the
plans to build a massive “international
Council of Foreign Ministers that meet
city” in an area of over 1,000 sq. km. on its
once a year, moves by rotation through
border with Assam. This project is known
members every two years.
as the Gelephu Project.
• Sri Lanka took charge as Chair this year
from Bangladesh, and India is Vice-Chair,
meaning that the troika of IORA is within
the South Asian region.

AMIGOS IAS 29
• While the IORA was formed in 1997 (then (1) I only (2) II only
called the Indian Ocean Region- (3) Both I and II (4) Neither I nor II
Association for Regional Cooperation) in Answer: 3
Mauritius, its genesis came from a speech Explanation:
Nelson Mandela gave in Delhi in 1995.
• President Droupadi Murmu has conferred
85. Which of the following pairs of places in the the National Award for Empowerment of
news and their countries are correctly Divyangjan 2023 to Sai Kaustuv Dasgupta
matched? of Satya Sai district.
I. Chashma: Pakistan About National award for Empowerment of
II. Newfoundland: Canada Person with Disabilities:
III. Kharson: Ukraine • Empowerment of persons with disabilities
Select the correct answer from the options is an inter-disciplinary process, covering
given below. various aspects namely, prevention, early
(1) Only I and II (2) Only II and III detection, intervention, education,
(3) Only III and I (4) All I, II, III vocational training, rehabilitation and
Answer: 4 social integration etc.
Explanation: • Apart from resources, it requires
• Chashma is located in Mianwali District dedicated efforts of persons and
near Kundian, Punjab, Pakistan. Hence, institutions involved in the process of
Pair I is correct. empowerment.
• Pakistan and China signed an MoU to build • In order to recognise their effort and
a new unit of the Chashma Nuclear Power encourage others to strive to achieve
Plant. excellence in this field, the National
Awards for Empowerment of Persons with
• Newfoundland is a province of Canada
Disabilities are being awarded every year.
composed of the island of Newfoundland.
Hence, Pair II is correct. • The National Awards for Empowerment of
Persons with Disabilities are being
• An underwater submersible “Titan” which
awarded by the Department of
was to travel to the site of the Titanic
Empowerment of Persons with
shipwreck on the ocean floor off
Disabilities, Ministry of Social Justice and
Newfoundland is missing.
Empowerment.
• Kherson is a port city in Ukraine that
• Till 2017, the Award Scheme was
serves as the administrative centre of
governed under the National Awards
Kherson Oblast. Hence, Pair III is correct.
Rules, 2013 which provided 7 categories
• The Kakhovka Dam in Ukraine was
of disabilities as per the Persons with
destroyed recently which has caused
Disabilities Act, 1995.
extensive flooding along the lower
• However, with the coming into force of
Dnieper River in Kherson and Kherson
the Rights of Persons with Disabilities Act,
Oblast region.
2016 with the effect from 19th April 2017,
86. Consider the following statements regarding
the number of specified disabilities
the National Award for Empowerment of
increased from 7 to 21 under the new Law.
Person with Disabilities.
• Accordingly, all the 21 disabilities have
I. These awards are presented by the
been included under the National Award
Minister for Social Justice.
Guidelines which have been notified in the
II. The National Award for Empowerment of Extraordinary Gazette of India dated 2nd
Divyangjan 2023 was awarded to Sai August 2018.
Kaustuv Dasgupta of Satya Sai district.
Source:
Which of the above given statements is/are https://www.deccanchronicle.com/natio
correct? n/in-other-news/031223/sai-kaustuv-
dasgupta-gets-presidents-

AMIGOS IAS 30
award.html#:~:text=Anantapur%3A%20Pr • Article 156. Term of Office of Governor (1)
esident%20Droupadi%20Murmu%20has,t The Governor shall hold office at the
he%20Wheelchair%20Warrior%20of%20I pleasure of the President.
ndia. Source:
87. Recently, Justice Syed Abdul Nazeer took the https://www.thehindu.com/news/nation
oath as Andhra Pradesh Governor. He will act al/andhra-pradesh/justice-syed-abdul-
as the Executive Head of State under? nazeer-takes-oath-as-andhra-pradesh-
(1) Article 153 (2) Article 154 governor/article66548303.ece
(3) Article 155 (4) Article 156 88. Which of the following organisations in
Answer: 2 Andhra Pradesh received the Digital
Explanation: Technology Sabha Award in February 2023?
• Former Supreme Court Judge Justice Syed (1) Andhra Pradesh State Skill Development
Abdul Nazeer, sworn in as Governor of Corporation
Andhra Pradesh, replacing Biswa Bhusan (2) Andhra Pradesh State Road Transport
Harichandan. Corporation
• AP High Court Chief Justice Prashant (3) Andhra Pradesh State Finance
Kumar Mishra administered the oath of Corporation
office to Justice Abdul Nazeer, at a (4) Andhra Pradesh Industrial Development
ceremony held at Raj Bhavan. Corporation
• Justice Abdul Nazeer was appointed as Answer: 2
Governor of Andhra Pradesh on February Explanation:
12. Born on January 5, 1958, in Beluvai, • Andhra Pradesh State Road Transport
Karnataka, he completed law at Sri Corporation (APSRTC) has received Digital
Dharmasthala Manjunatheshwara Law Technology Sabha Award in the category
College, Mangalore, and practiced in of “Internet of Things”. RTC Vice-
Karnataka High Court and other courts for Chairman and Managing Director RP
about 20 years. Thakur received the award from the RTC
• On February 17, 2017, Justice Abdul House virtually.
Nazeer was elevated to the Supreme • We won this award by competing with
Court. After serving for almost six years in many government institutions at national
the Apex Court, he retired on January 4, level. The Corporation has been
this year. implementing all types of digital payments
• Article 153. Governors of States: There like UPI,QR code, net banking, credit/debit
shall be a governor for each state. cards and e-wallet on our mobile app. Seat
[Provided that nothing in this article shall inventory of APSRTC was made available
prevent the appointment of the same on all online travel agency portals and
person as governor for two or more Apps like redBus, MakeMyTrip, Goibibo,
states.] ixigo and AbhiBus,” Thakur said. He said
• Article 154: Executive Power of the State the APSRTC has provided 2,500 ticket
(1) The executive power of the state shall booking agent networks in Andhra
be vested in the governor and shall be Pradesh, Telangana, Karnataka, Tamil
exercised by him either directly or through Nadu and Chhattisgarh.
officer’s subordinate to him in accordance • As part of Digital India programme,
with this Constitution. APSRTC is mulling a new project (Unified
• Article 155: Appointment of Governor The Ticketing Solution) in which passengers
governor of a state shall be appointed by can avail all types of services of the
the president by warrant under his hand Corporation like ticket booking and
and seal. cancellation, prepone/postponement of
journey, avail bus pass, live track buses
and send a parcel or courier on a single

AMIGOS IAS 31
mobile App. All the above services will be • NQAS are currently available for District
available through digital payments. After Hospitals, CHCs, PHCs and Urban PHCs.
the introduction of the new project, the • Standards are primarily meant for
digital transaction share will go up to 50 providers to assess their own quality for
per cent. improvement through pre-defined
Source: https://www.sakshi.com/telugu- standards and to bring up their facilities
news/andhra-pradesh/apsrtc-received- for certification.
digital-technology-sabha-award-5th-time- • The National Quality Assurance Standards
1530238 are broadly arranged under 8 "Areas of
89. Which one of the following sport/games is Concern"– Service Provision, Patient
not part of Aadudam Andhra programme? Rights, Inputs, Support Services, Clinical
(1) Cricket (2) Football Care, Infection Control, Quality
(3) Volleyball (4) Kabaddi Management and Outcome.
Answer: 2 • These standards are ISQUA accredited and
Explanation: meets global benchmarks in terms of
comprehensiveness, objectivity, evidence
and rigor of development.
Source:
https://www.thehansindia.com/andhra-
pradesh/vizianagaram-25-phcs-get-nqas-
recognition-
842216#:~:text=Vizianagaram%3A%20Ar
ound%2025%20primary%20health,provid
ing%20services%20under%20their%20jur
isdiction.
91. “The Maitree Super Thermal Power Project”
is being developed by:
(1) Sri Lanka - India Friendship Power
Company
90. Recently, 25 primary health centres of which (2) Bangladesh India Friendship Power
district have been recognised by the Union Company
government and received National Quality
(3) Nepal India Friendship Power Company
Assurance Standards (NQAS) certification?
(4) Indonesia and India Friendship Power
(1) Vizianagaram (2) Srikakulam
Company
(3) Guntur (4) Kurnool
Answer: 2
Answer: 1
Explanation:
Explanation:
 The Maitree Super Thermal Power Project
National Quality Assurance Standards is being developed by Bangladesh India
• Around 25 primary health centres (PHCs) Friendship Power Company, a 50:50 joint
from the Vizianagaram district have been venture between India’s National Thermal
recognised by the Union government and Power Corporation & Bangladesh Power
received National Quality Assurance Development Board.
Standards (NQAS) certification. 92. “Hiroshima AI Process (HAP)” was initiated
About National Quality Assurance Standards under the aegis of
(NQAS): (1) G20 (2) BRICS
• National Quality Assurance Standards (3) ASEAN (4) G7
have been developed keeping in view of
Answer: 4
specific requirements for public health
facilities as well global best practices. Explanation:

AMIGOS IAS 32
• The annual G-7 summit took place in • Green Climate Fund aims to assist
Hiroshima from 19 to 21 May 2023. The developing countries in adaptation and
Hiroshima AI Process was initiated during mitigation practices to counter climate
the summit. change. Hence, Statement I is correct.
• The annual G-7 summit was hosted by • The Green Climate Fund was established
Japan in Hiroshima from 19 to 21 May in 2010 as a part of the UNFCCC’s financial
2023. framework to channel funds from richer
• During the summit, the Hiroshima AI nations to developing countries in order to
Process (HAP) was initiated. help them reduce climate change and
• HAP is an effort to determine a way adapt to its effects. Hence, Statement II is
forward to regulate Artificial Intelligence not correct.
(AI). 94. Consider the following statements with
• The ministerial declaration of the G-7 reference to the United Nations Security
Digital and Tech Ministers’ Meeting (April Council (UNSC):
2023) discussed “responsible AI” and I. The UNSC is one of the six principal organs
committed to promoting human-centric of the United Nations.
and trustworthy AI. II. It has 15 members, and each member has
• Despite deliberations and accords to two votes.
regulate AI, each country is promoting III. The UNSC can impose sanctions or
human-centric and trustworthy AI. authorise the use of force to maintain or
• Hiroshima AI Process: restore international peace and security.
• Despite several other issues, the Which of the statements given above is/are
communiqué accorded more importance correct?
to AI. It discussed inclusive AI governance (1) Only I (2) Only II
and trustworthy AI, in line with shared (3) Only I and III (4) All the above
democratic values. Answer: 3
• It also took stock of the opportunities and Explanation:
challenges of generative AI.  The UNSC has 15 members with one vote
• The G-7 communiqué aimed to establish each, and it can resort to imposing
the Hiroshima AI process, through a G7 sanctions or authorising the use of force
working group, in an inclusive manner and to maintain or restore international peace
in cooperation with the OECD and GPAI, and security. The UNSC is one of the six
for discussions on generative AI by the end principal organs of the United Nations.
of 2023. 95. What is the theme of the Ninth P20 Summit,
93. Which of the following statements regarding held in New Delhi on October 13–14, 2023?
the ‘Green Climate Fund’ is/are correct? (1) Parliaments for Global Peace and Security
I. It is intended to assist developing (2) Parliaments for One Earth, One Family,
countries in adaptation and mitigation One Future
practices to counter climate change.
(3) Parliaments for Sustainable Development
II. It was founded under the aegis of UNEP,
(4) Parliaments for Democracy and Good
OECD, the Asian Development Bank, and
Governance
the World Bank.
Answer: 2
Select the correct answer from the options
given below. Explanation:
(1) Only I (2) Only II The Ninth P20 Summit will have the theme
“Parliaments for One Earth, One Family,
(3) Both I and II (4) Neither I Nor II
One Future.” This theme is inspired by the
Answer: 1 ancient Indian philosophy of Vasudhaiva
Explanation: Kutumbakam, which means “The World is
One Family.”

AMIGOS IAS 33
96. Recently, Greater Vishakhapatnam • And in this regard, the Olive Ridley Turtle
Municipal Corporation and the Forest Conservation festival is celebrated with
Department collaborated to conserve which great vigor.
of the following species? Activities:
(1) Indian Star Tortoise • Observe the Olive Ridley’s laying eggs
(2) Tiny Grass Blue • Help the volunteers to release hatchlings
(3) Marsh Crocodile into the sea.
(4) Olive Ridley Turtle • Tourists can see different species of
Answer: 4 turtles.
Explanation: 97. Consider the following Cities of Andhra
• Greater Vishakhapatnam Municipal Pradesh.
Corporation aims to collaborate with the I. Visakhapatnam
forest department to develop an Olive II. Tirupati
Ridley Turtle Conservation Centre. III. Amaravati
About Olive Ridley Turtles: IV. Anantapuram
• The olive Ridley gets its name from the Which of the above are selected under Smart
olive green colour of its heart-shaped Cities Mission of India?
shell. (1) Only I and III (2) Only II and III
• The species is among the smallest of the (3) Only I, II and III (4) All the above
world’s sea turtles and is found primarily Answer: 3
in the tropical regions of the Pacific,
Explanation:
Indian, and Atlantic oceans.
• Four cities of Andhra Pradesh —
• These turtles are carnivores and get their
Visakhapatnam, Tirupati, Kakinada, and
name from their olive-colored carapace.
Amaravati — were picked for the Union
• They are best known for their unique mass government’s Smart Cities Mission (SCM)
nesting called Arribada, where thousands between 2016 and 2017. Six years since
of females come together on the same the launch of the ambitious project, the
beach to lay eggs. progress across these cities paints a mixed
• The species is listed as Vulnerable in the picture. Overall, projects worth 4,700
IUCN Red List, Appendix 1 in CITES, and crore have been completed out of the
Schedule 1 in Wildlife Protection Act, planned 6,900 crore in the four cities.
1972. • APCRDA allocates Rs 40 crore for
About Olive Ridley Turtle Conservation: developing Amaravati as a Smart city at
• Olive Turtle conservation project started par with top 100 cities in the country
in 2017. under CITIIS (City Investments to Innovate
• During December to March, many Olive Integrated and Sustain) → 15 E-health
Ridley turtles come to the western coast centres to be set up in Amaravati.
of Maharashtra and lay their eggs. Source:
• These eggs are then guarded by a group of https://www.newindianexpress.com/stat
volunteers against predators and es/andhra-pradesh/2023/feb/04/15-e-
poachers. health-centres-to-be-set-up-in-andhra-
• Eggs naturally hatched within 40 - 45 days pradeshs-
and the hatchlings will then be released to amaravati2544282.html#:~:text=GUNTUR
the sea. %3A%20Andhra%20Pradesh%20Capital%
• Previously they were either sold in the 20Region,100%20cities%20in%20the%20
markets or were destroyed by dogs, but country.
now these conversational efforts helped
the rise of the Ridley family.

AMIGOS IAS 34
98. Andhra Pradesh Forest Department aimed to 99. How many of the tourist police stations have
develop how many Nagaravanams in urban been launched by the Andhra Pradesh Chief
areas across the state.? Minister recently across the state at
(1) 80 (2) 90 important religious and tourist places?
(3) 100 (4) 120 (1) 15 (2) 20
Answer: 3 (3) 25 (4) 30
Explanation: Answer: 2
• Andhra Pradesh Forest Department Explanation:
officials are gearing up to develop over • Giving a boost to tourism security in
100 Nagaravanams in urban areas across Andhra Pradesh, Chief Minister Y.S. Jagan
the state. Mohan Reddy has inaugurated Tourist
About Nagaravanam Initiative: Police Stations at 20 important religious
• The aim of the scheme is to provide and tourist places in various districts
wholesome living environment to urban virtually from his Camp Office at Tadepalli.
areas and thereby contributing to the • RK Beach in Visakhapatnam,
creation of smart, clean, green and • Sri Kodanda Rama Swamy temple at
healthy cities. Vontimitta in YSR district,
• During 2016-17 Ministry of Environment, • Kukkuteswara Swamy temple at
Forest & Climate Change, Government of Pithapuram in Kakinada district,
India have conceptualized the programme • Pushkar Ghat at Rajahmundry,
for climate smart green cities by name • Sri Venkateswara Swamy temple at
Nagarvan – Udyan Yojana. Dwaraka Tirumala in Eluru district
• It is (Ek Kadam Hariyaliki) to • Manginapudi Beach and the Mopidevi
develop/create at least one city Forest in Temple in Krishna district,
each city over an extent of 25 ha. – 100 ha. • Indrakeeladri Temple and Bhavani Island
within the adjacent Forest areas. in NTR district,
• In continuation of the Nagarvan Udyan • Mypadu Beach and the Sri Penusila
Yojana Scheme, State Government of Lakshmi Narasimha Swami temple at
Andhra Pradesh has taken initiative for Penchalakona in SPSR Nellore district,
further development of Nagaravanams
• Mahanandi and Ahobilam temples in
across the state as urban lung spaces and
Nandyala district,
started a new scheme called
• Raghavendra Swamy temple at
“Nagaravanam” as state development
Mantralayam in Kurnool district,
programme.
• Horsley Hills in Annamayya district and
• As part of AP Eco-Tourism Project, over 44
Nagaravanams are being developed • Lepakshi temple in Satya Sai district.
across the State, including in: Source:
Rayadurgam, Kalyanadurgam, https://www.deccanchronicle.com/natio
Puttaparthi, Pampanuru, Hindupur, and n/current-affairs/140223/cm-
Palmaneru in Ananthapur circle, inaugurates-tourist-police-stations-at-20-
Kondaveedu, Perecherla, Darsi-I, Darsi-II, centres.html
Perlavanipalem, Naidupeta, Kotturu in 100. Consider the following dams.
Guntur circle, Potlampally, I. Paleru dam
Gargeyapuram, Dhone in Kurnool circle, II. Gollaleru dam
Kothuru-Tadepalli, Krishnalanka, Butterfly III. Thimmaraju dam
Park, and Diwancheruvu in Which of the above given dams form part of
Rajamahendravaram, Divyaramam, RS Owk Tunnel Project?
park in Tirupati, and Saripalli in
(1) I only (2) I and II only
Visakhapatnam.
(3) II and III only (4) All the above
Answer: 4

AMIGOS IAS 35
Explanation:  The EU-India Trade and Technology
• Andhra Pradesh Chief Minister Council is the second bilateral forum for
Inaugurated the Owk Tunnel Project the EU and the first one established with
About Owk Tunnel Project: any partner for India. The EU and the US
• Three earth dams comprising of Paleru, launched a TTC in June 2021.
Gollaleru and Thimmaraju form the Owk 102. Which of the following statements is/are
Reservoir complex. correct with reference to the International
• It is a balancing reservoir in Srisailam Right Labour Organization (ILO)?
Bank Canal & Galeru-Nagari Sujala I. The ILO is a tripartite U.N. agency,
Sravanthi systems. founded in 1919, bringing together
• Previously the Paleru and Thimmaraju governments, employers, and workers of
earth dams are the tanks which feeds member states to promote decent work.
water to Owk village ayacut of II. India is a founding member of the ILO.
approximately 600 acres and 2000 acres III. India has ratified all of its eight
respectively core/fundamental ILO conventions.
• The Stage I of Owk reservoir was taken up Select the correct answer using the code given
as part of the SRBC Scheme under World below:
Bank aided programme (1) Only I and II (2) Only II
• To raise the bund level to +224.00 M and (3) Only III (4) All the above
to impound water to a level of 220.00 M Answer: 1
with a capacity of 1.91 TMC to feed water Explanation:
to SRBC ayacut from Blocks 12 to 21.
 The ILO is a tripartite U.N. agency, and
• It is to stabilse for an ayacut of around
India, a founding member, has ratified six
80,000 acres. The work was taken up and
out of the eight core/fundamental ILO
completed.
conventions.
• The Stage II programme was taken up as
103. Which of the following countries and
part of GNSS system raising the level of
organizations are part of the “Black Sea Grain
the Owk Reservoir complex to +231.00 M
Initiative”?
to impound water up to +227.00 M level
(1) Ukraine, Russia, Turkey, Food and
with a capacity of 4.148 TMC and the work
Agricultural Organization
is also completed.
(2) Ukraine, Russia, Turkey, and the United
Source:
Nations
https://www.thehindu.com/news/nation
al/andhra-pradesh/andhra-pradesh-chief- (3) Ukraine, Russia, United States and Food
minister-inaugurates-tunnel-ii-of-owk- and Agricultural Organization
project/article67590683.ece (4) Ukraine, Russia, Turkey, and the European
101. Which of the following statements with Union
respect to the EU-India Trade and Technology Answer: 2
Council is/are correct? Explanation:
I. It is the second bilateral forum for the EU. • The Black Sea Grain Initiative was
II. It is the first bilateral forum established launched by Ukraine, Russia, Turkey, and
with any partner for India. the United Nations.
Select the correct answer using the code given • It aims to facilitate the export of grain
below: from Ukraine through the Black Sea.
(1) Only I (2) Only II 104. “Mouvement du 23 Mars (M23)” is an
(3) Both I and II (4) Neither I nor II insurgent group active in which of the
following countries?
Answer: 3
(1) Tunisia (2) Nigeria
Explanation:
(3) Democratic Republic of Congo
(4) South Sudan

AMIGOS IAS 36
Answer: 3 Explanation:
Explanation: • Andhra Pradesh Fisheries and Animal
 Mouvement du 23 Mars (M23) is an Husbandry Minister said that the first
insurgent group active in the Democratic cargo ship will berth at Mulapeta Port in
Republic of Congo. It is active in eastern Srikakulam district September 2024.
DRC leading to clashes with the Congolese About Mulapeta Port:
army. • The Mulapeta Port, being built in an area
105. Exercise ‘Al Najah’ Exercise ‘Eastern Bridge’ of 1250 acre with an annual handling
and Exercise ‘Naseem Al Bahr’ are joint capacity of 23.5 million tonnes in the
military exercises between India and Srikakulam District.
(1) Saudi Arabia • This port will have four loading and
(2) United Arab Emirates unloading berths to handle exports and
(3) Maldives imports of coal, multipurpose containers,
(4) Oman and general cargo.
Answer: 4 • This port is expected to provide 25,000
jobs and play a key role in handling
Explanation:
exports and imports from states like
 Joint Military Exercises between India and Chhattisgarh, Jharkhand, Madhya
Oman are Al Najah (Army exercise), Pradesh, and Odisha.
Eastern Bridge (Air Force exercise), and
Source:
Naseem Al Bahr (Naval Exercise).
https://infra.economictimes.indiatimes.c
106. Which of the following organisations aims to om/news/ports-shipping/andhra-cm-lays-
create 12,000 “Energy clubs” in Andhra foundation-stone-for-mulapeta-port-in-
Pradesh to help students understand the srikakulam/99614705
importance of energy conservation and
108. The Andhra Pradesh Cabinet gave its
efficiency?
approval for the establishment of the
(1) Energy Efficiency Services Limited National Law University in the state. It is
(2) Bureau of Energy Efficiency establishing in?
(3) AP Power Generation Corporation (1) Anantapur (2) Kadapa
(4) International Energy Agency (3) Kurnool (4) Amaravati
Answer: 2 Answer: 3
Explanation: Explanation:
• The Bureau of Energy Efficiency (BEE) aims • The Andhra Pradesh Cabinet gave its
to create 12,000 Energy Clubs in Andhra approval for the establishment of the
Pradesh to help students understand the National Law University in the state.
importance of energy conservation and • The university, the second of its kind in the
efficiency. country, will come up on a 50-acre campus
Source: at Kurnool.
https://www.thehindu.com/news/nation Source: https://news.abplive.com/andhra-
al/andhra-pradesh/bureau-of-energy- pradesh/andhra-cabinet-s-nod-for-
efficiency-aims-to-create-12000-energy- national-law-university-at-kurnool-
clubs-in-andhra- 1581031
pradesh/article66462669.ece
109. “E-MIRCHA project” that was initiated by the
107. Mulapeta Port is located in which district? AP Horticultural Department. It is being
(1) Parvathipuram Manyam implemented in coordination with?
(2) Srikakulam (1) Digital Green NGO
(3) Vizianagaram (2) Bill and Milinda Gates Foundation
(4) Vishakhapatnam (3) Both (1) and (2)
Answer: 2 (4) Neither (1) nor (2)

AMIGOS IAS 37
Answer: 3 III. Multilateral Investment Guarantee
Explanation: Agency (MIGA)
• Nearly 49,000 chilli farmers had received IV. International Finance Corporation (IFC)
various benefits under the E-MIRCHA V. International Centre for Settlement of
project that was initiated by the AP Investment Disputes (ICSID)
government. In how many of the above constituents of the
• Andhra Pradesh is the major chili- World Bank Group is India a member?
producing State in India.
• AP government- Horticulture department (1) Only two (2) Only three
in coordination with Digital Green NGO (3) Only Four (4) All
including Bill and Milinda Gates Answer:3
Foundation has introduced the E-MIRCHA
Explanation:
project to increase the quality of mirchi
• The World Bank Group is one of the
produced in the state.
world’s largest sources of funding and
Source:
knowledge for developing countries. Its
https://www.newindianexpress.com/stat
five institutions share a commitment to
es/andhra-pradesh/2023/feb/13/49000-
reducing poverty, increasing shared
chilli-farmers-of-andhra-benefit-under-e-
prosperity, and promoting sustainable
mircha-project-
development.
2546951.html#:~:text=The%20main%20i
• India is a member of four of the five
nitiative%20of%20this,production%20cap
constituents of the World Bank Group viz.,
acity%20of%20the%20farmers.&text=Ima
the International Bank for Reconstruction
ge%20used%20for%20representational%
and Development (IBRD), International
20purpose%20.
Development Association (IDA),
110. 2023 Edition “Global Tech Summit” was held
International Finance Corporation (IFC),
in?
and Multilateral Investment Guarantee
(1) Vijayawada (2) Visakhapatnam Agency (MIGA).
(3) Tirupati (4) Kakinada • India is not a member of ICSID
Answer: 2 (International Centre for Settlement of
Explanation: Investment Disputes).
• The co-convenor of the Global Tech • India has been accessing funds from the
Summit and CEO of Pulsus Company World Bank (mainly through IBRD and
Gedela Srinubabu said that European IDA) for various development projects.
Business Technology Center (EBTC) would 112. This foreign policy of the Indian government
set up a center of excellence in Vizag as AP is an initiative to strengthen economic and
is an agricultural state security cooperation with neighbouring
• Above 1,000 delegates from 25 countries maritime countries. It was introduced in
such as the UK, USA, Europe and also from response to the harsh reality that instability
African countries would take part in it. in the marine environment anywhere would
Source: not enhance India's maritime security and its
https://timesofindia.indiatimes.com/city/ national interests. This policy demonstrates
visakhapatnam/two-day-global-tech- the linkage between maritime cooperation,
summit-from- maritime security, and economic
today/articleshow/97965694.cms development. It brought to focus the
111. Consider the following organisations. increasingly critical role that the Coast Guard
I. International Development Association agencies of the Indian Ocean littorals will
(IDA) play in ensuring the security of their
II. International Bank for Reconstruction and respective vast EEZs for facilitating economic
Development (IBRD) activities based on the blue economy.

AMIGOS IAS 38
Which of the following Indian Foreign Policy is Select the correct answer using the options
described above? given below:
(1) Malabar Exercise (1) Only I (2) Only II
(2) SAGAR Doctrine (3) Both I and II (4) Neither I Nor II
(3) Quad Grouping Answer: 2
(4) Act East Policy Explanation:
Answer: 2 • Currently, India and Nepal have border
Explanation: disputes over the Kalapani-Limpiyadhura-
• The Security and Growth for All in the Lipulekh trijunction between India-Nepal
Region (SAGAR) Doctrine is an initiative and China and the Susta area (West
introduced by the Modi government to Champaran district, Bihar).
strengthen economic and security Kalapani Region:
cooperation with the maritime neighbor • Kalapani is a valley that is administered by
countries. India as a part of the Pithoragarh district
Salient features of SAGAR doctrine: of Uttarakhand. It is situated on the
• SAGAR doctrine effectively demonstrates Kailash-Mansarovar route. Hence,
the linkages between maritime Statement I is incorrect.
cooperation, maritime security, and • Kalapani is advantageously located at a
economic development. height of over 20,000 feet and serves as an
• Defence and security relations are observation post for that area.
essential pillars of bilateral cooperation • The Kali River in the Kalapani region
with some of the IOR countries like the demarcates the border between India and
Seychelles, Mauritius, and the Maldives. Nepal. Hence, Statement II is correct.
• In this doctrine, India intended to expand • The Treaty of Sugauli, signed by the
strategic partnerships with other IOR Kingdom of Nepal and British India (after
littorals in Asia and Africa, and SAGAR the Anglo-Nepalese War) in 1816, located
provided the mechanism to do so. the Kali River as Nepal's western boundary
• SAGAR stressed the critical role of IONS with India.
(Indian Ocean Naval Symposium), an • The discrepancy in locating the source of
Indian initiative that had 35 navies as its the Kali river led to boundary disputes
members; and IORA (Indian Ocean Rim between India and Nepal, with each
Association). country producing maps supporting their
• After SAGAR, India actively continued to own claims.
pursue its program of capacity-building
initiatives in the IOR (Indian Ocean
Region).
• SAGAR brought to focus the increasingly
critical role that the Coast Guard agencies
of the Indian Ocean littorals will play in
ensuring the security of their respective
vast EEZs for facilitating economic
activities based on the blue economy.
113. Which of the following statements with
respect to the Kalapani dispute between
India and Nepal is/are correct?
I. Kalapani valley is located in Uttara
Pradesh.
II. The Kali river demarcates the border
between India and Nepal in this region.

AMIGOS IAS 39
114. The recent first ‘India-EU Joint Naval Exercise’ III. Vedic students are also covered under this
was held in? scheme.
(1) The Mediterranean Sea Which of the above given statements is/are
(2) The Gulf of Guinea incorrect?
(3) The Indian Ocean (1) I only (2) I and II only
(4) The Baltic Sea (3) II and III only (4) All the above
Answer: 2 Answer: 3
Explanation: Explanation:
The European Union (EU) and India conducted • Andhra Pradesh Brahmin Welfare
their first joint naval exercise in the Gulf of Corporation announced that they will be
Guinea. sending a proposal to the state
115. Consider the following statements with government to include Vedic students
respect to LiFE. under the Amma Vodi scheme.
I. The LiFE initiative was launched by Prime About Jagananna Amma Vodi:
Minister Narendra Modi at COP27 of • This scheme was launched in the year
UNFCC. 2020 by the Chief Minister of Andhra
II. “LiFE Lessons from India report” was Pradesh.
released by UNEP. • This scheme provides poor mothers (or
Which of the above statements is/are correct? recognized guardian in the absence of
(1) Only I (2) Only II mother) ₹15,000/- per annum for sending
their children to school.
(3) Both I and II (4) Neither I Nor II
• The scheme is managed by the School
Answer: 4
Education Department, Government of
Explanation:
Andhra Pradesh.
• The LiFE initiative was launched by Prime
• The objective of the scheme is to enhance
Minister Narendra Modi at COP26 in
access to schools, ensure equity, assuring
Glasgow in 2021.
quality education, improve attendance,
• It aims to encourage the adoption of and retention, achieve better learning
sustainable lifestyles in India and outcomes, and for the overall
internationally to tackle the challenges of development of the child from Classes 1st
environmental degradation and climate to 12th.
change.
• The scheme is expected to lead to a strong
• LiFE Lessons from India report was foundation for the increase in the overall
released by International Energy Agency. Gross Enrolment Ratio (GER) at the
• As per the report the ‘Lifestyle for primary and secondary levels of education
Environment’ or LiFE initiative of India for of the state including Higher Education
the adoption of energy-efficient and framed Guidelines for
behaviour can save the world $440 billion implementation of the program.
in 2030. Benefits of the scheme:
• It would be equivalent to around 5% of all • The eligible mothers/guardians will be
fuel spending across the global economy sanctioned ₹ 14000/- which will be
in 2030. deposited directly into the beneficiary’s
116. Consider the following statements regarding savings account of the Nationalized Bank /
the Jagananna Amma Vodi scheme: Post Office in January every year until the
I. This scheme provides BPL category child passes Class 12th.
mothers with ₹15,000 per annum for • An amount of ₹ 1000/- from each
sending their children to school. beneficiary shall be deducted at the
II. The scheme covers students studying source and released to School/College
from 1st to 10th grade in government Committee as District Toilet Maintenance
schools.

AMIGOS IAS 40
Fund (DTMF) under the control of the Digital Mission (ABDM). AP stood first in
District Collector. linking the maximum number of health
Eligibility of the scheme: records between October 20, 2022, and
• Must be legal resident of AP. December 20, 2022, per 1 lakh population.
• Must be from a BPL household with a • Eluru, Visakhapatnam, and Palnadu stood
white ration card. in the top three places in linking maximum
• The student must be studying in a health records to ABDM. AP also achieved
government school/ junior college an award for the maximum number of
between class 1 and 12. records linked to ABHA by an integrator.
• If child discontinue the study in middle of Source:
the session of academic year, then he will https://timesofindia.indiatimes.com/city/
not be able to avail the benefits of the vijayawada/ap-bags-5-awards-under-
scheme. abdm/articleshow/97336700.cms
117. NTPC is to set up a "New Energy Park" in 119. The three-day 16th Global Health Summit,
Andhra Pradesh to produce products out of organised by the American Association of
green hydrogen, green ammonia and green Physicians of Indian Origin (AAPI), called for a
methanol at a cost of Rs 1,10,000 crore in two new global health architecture that gives
phases. It is establishing in? priority to promoting health and preventing
disease rather than only treating the sick. It
(1) Kurnool District
was held in?
(2) Anakapalli District
(1) Vijayawada (2) Tirupati
(3) Bapatla District
(3) Visakhapatnam (4) Rajahmundry
(4) Konassema District
Answer: 3
Answer: 2
Explanation:
Explanation:
• The three-day 16th Global Health Summit
• NTPC is to set up a "New Energy Park" at
organised by American Association of
Pudimadaka in Anakapalli district of
Physicians of Indian Origin (AAPI) in
Andhra Pradesh to produce products out
Visakhapatnam called for a new global
of green hydrogen, green ammonia and
health architecture that gives priority to
green methanol at a cost of Rs 1,10,000
promoting health and preventing disease,
crore in two phases.
rather than only treating the sick.
Source: https://www.business-
• The architecture must be based on
standard.com/article/companies/andhra-
common vision, be inclusive, and
govt-clears-rs-1-10-trn-proposal-of-ntpc-
stimulate innovative solutions to health
for-new-energy-park-
challenges.
123020701823_1.html#:~:text=NTPC%20
• The summit also discussed many
will%20set%20up%20New,55%2C000%20
important issues related to Andhra
crore%2C%20the%20release%20said.
Pradesh. Taking forward the call of Chief
118. Andhra Pradesh won five national-level
Minister Y.S. Jagan Mohan Reddy, AAPI
awards under the Ayushman Bharat Digital
decided to provide mental healthcare for
Mission (ABDM). AP stood in which rank for
school students to prevent suicides. For
linking the number of health records
this purpose, a pilot project is being
between October 20, 2022, and December
launched to train 10,000 teachers as
20, 2022, per 1 lakh population?
counselors.
(1) 1st (2) 2nd
Source :
(3) 3rd (4) 4th https://www.deccanchronicle.com/natio
Answer: 1 n/in-other-news/080123/vizag-summit-
Explanation: calls-for-new-global-health-
• Andhra Pradesh won five national-level architecture.html
awards under the Ayushman Bharat

AMIGOS IAS 41
120. Consider the following statements with ?PRID=1912123#:~:text=The%20Second%
reference to G20 meetings. 20G20%20Infrastructure%20Working,Vis
I. The second G20 Infrastructure Working akhapatnam%20on%20March%2029th%2
Group (IWG) meeting was held in C%202023.
Visakhapatnam. 121. Consider the following statements regarding
II. The theme of the meeting was “Financing the Henley Passport Index:
Cities of Tomorrow: Inclusive, Resilient, I. It ranks passports based on the number of
and Sustainable." countries their holders can access without
Which of the above statements is/are correct? a prior visa.
(1) I Only (2) II Only II. In 2023, India’s ranking on the Passport
(3) Both I and II (4) Neither I Nor II Index declined to 80th place.
Answer: 3 How many of the statements given above
Explanation: is/are correct?
• The Second G20 Infrastructure Working (1) Only I (2) Only II
Group (IWG) meeting under the Indian (3) Both I and I (4) Neither I Nor II
G20 Presidency concluded successfully in Answer: 3
Visakhapatnam on March 29th, 2023. The Explanation:
two-day meeting was co-chaired by • In 2023, India’s ranking on the Passport
Department of Economic Affairs, Ministry Index improved to 80th place.
of Finance, Government of India, along • Singapore is currently the most powerful
with Australia and Brazil. 57 delegates passport in the world, allowing visa-free
from 14 G20 member countries, 8 access to 192 travel destinations.
invitees, and 10 international 122. Which of the pairs of defense exercises
organizations attended the meeting between India and countries involved is
physically. correctly matched?
• The G20 Infrastructure Working Group I. Ekuverin: Maldives
deliberates on various aspects of
II. Khaan Quest: Mongolia
infrastructure investments, including
III. Lamitye: Sri Lanka
developing infrastructure as an asset
class, promoting quality infrastructure IV. Dustlik: Kazakhstan
investment, and identifying innovative Which of the pairs given above are correctly
instruments for mobilizing financial matched?
resources for infrastructure investment. (1) Only I and IV (2) Only I and II
• The agenda for the meeting involved (3) Only II and III (4) Only III and IV
discussions on the ways to augment Answer: 2
financing in cities of tomorrow, along with Explanation:
other priorities outlined in the 2023 • Ekuverin: India and Maldives
Infrastructure Agenda. • Khaan Quest: Annual military exercise in
• During the discussion, members discussed Mongolia
and agreed on the way forward for • Lamitye: India-Seychelles joint military
important deliverables for the year, exercise
including the flagship theme of “Financing
• Dustlik: India and Uzbekistan
Cities of Tomorrow- Inclusive, Resilient
123. “The Shangri-La Dialogue (SID)” is an inter-
and Sustainable”. On this agenda,
governmental security forum held annually
members discussed various facets of
by an independent think tank in?
making cities economic centres of growth,
financing urban infrastructure, building (1) Malaysia (2) Singapore
future-ready urban infrastructure etc. (3) Philippines (4) Australia
Source: Answer:2
https://pib.gov.in/PressReleasePage.aspx Explanation:

AMIGOS IAS 42
• The International Institute for Strategic I. It includes India, the UAE, Saudi Arabia,
Studies (IISS) Shangri-La Dialogue is Asia’s the European Union, France, Italy,
premier defence summit. Germany, and the US.
• It’s a unique meeting where ministers II. The project is part of the Partnership for
debate the region’s most pressing security Global Infrastructure Investment (PGII).
challenges, engage in important bilateral III. It is part of China’s Belt and Road Initiative
talks and come up with fresh approaches (BRI).
together. Which of the statements given above are
• The 2023 event took place place in correct?
Singapore on 2–4 June. (1) Only I (2) Only II
(3) Only II and III (4) All the above
Shangri La Dialogue: Answer: 3
• The Shangri-La Dialogue (SID) is an inter- Explanation:
governmental security forum held India – Middle East – Europe mega economic
annually by an independent think tank, corridor:
the International Institute for Strategic
• A Memorandum of Understanding (MoU)
Studies (IISS).
was signed between the governments of
• It is attended by defence ministers, India, the US, Saudi Arabia, the European
permanent heads of ministries, and Union, the UAE, France, Germany, and
military chiefs of 28 Asia-Pacific states. Italy to establish the IMEC.
• The forum gets its name from the Shangri- • IMEC is part of a broader initiative called
La Hotel in Singapore where it has been the Partnership for Global Infrastructure
held since 2002. Investment (PGII).
• The Shangri-La Dialogue was conceived by • The PGII was initially introduced during
the current IISS Director-General and the G7 summit in the UK in June 2021.
Chief Executive Sir John Chipman in 2001
• The project aims to establish a network of
in response to the clear need for a forum
transport corridors, including railways and
where the Asia Pacific defence ministers
sea lanes.
could engage in dialogue aimed at building
• IMEC is seen as a response to China’s Belt
confidence and fostering practical security
and Road Initiative (BRI), providing an
cooperation.
alternative infrastructure network.
124. Which of the following organizations /
institutions releases the Global Findex
Report?
(1) Economist Intelligent Unit
(2) International Monetary Fund
(3) World Bank
(4) World Economic Forum
Answer:3
Explanation:
• The Global Findex Report is released by
the World Bank.
• It provides data on financial inclusion and
access to financial services worldwide.
125. Consider the following statements with 126. Which of the following statements is correct
reference to the India-Middle East-Europe with reference to “AMPHEX 2023”?
mega economic corridor project: I. It is a biennial tri-services exercise.
II. It was conducted in Kakinada, Andhra
Pradesh.

AMIGOS IAS 43
III. It is the first time that the exercise has Explanation:
been undertaken at Kakinada. • Proving that people are living in Andhra
Select the correct answer using the codes Pradesh with ease, as many as three cities
given below. from the state secured top spots in the
(1) Only I and II (2) Only II and III Citizen Perception Survey.
(3) Only I and III (4) All the above • According to the official reports, three
Answer: 4 cities from Andhra Pradesh, including
Explanation: Guntur, Vijayawada, and Visakhapatnam,
secured positions in the top ten list in the
• The biennial Tri-Services Amphibious
national-level survey.
Exercise, AMPHEX 2023, was conducted in
Kakinada, Andhra Pradesh, from January • Meanwhile, Bengaluru topped the overall
17 to 22. AMPHEX is aimed at joint training national rankings.
of elements of all three services in various • Guntur has reported the highest
facets of amphibious operations to percentage of the participation of city
enhance interoperability and synergy. population in the survey(51.37%) in the
• AMPHEX 23 is the first time that the entire country, while Vijayawada reported
exercise was undertaken at Kakinada and 32.12% and Visakhapatnam 16.72%.
was the largest ever AMPHEX conducted • The Citizen Perception Survey conducted
till date. by the Ministry of Housing and Urban
• The participating forces undertook Affairs, Government of India, is part of the
complex exercises in all domains of Ease of Living Index, which seeks to
amphibious operations over five days. directly capture the perceptions of
citizens with respect to the quality of life
• The exercise witnessed the participation
in their cities.
of a number of amphibious ships,
consisting of Large Platform Docks (LPD), • The survey is conducted every year on
Landing Ships and Landing Crafts, Marine various parameters like city development,
Commandos (MARCOS), helicopters, and basic amenities, sanitation, and other
aircraft from the Indian Navy. aspects, provided by over 1,800 local
bodies.
• The Indian Army participated in the
exercise with over 900 troops, which Source:
included Special Forces, artillery, and https://www.newindianexpress.com/stat
armored vehicles. Jaguar fighters and C- es/andhra-pradesh/2023/jan/09/three-
130 aircraft from the IAF also participated ap-cities-top-in-ease-of-living-survey-
in the exercise. 2536009.html#:~:text=According%20to%
20the%20official%20reports,in%20the%2
Source:
0national%20level%20survey.
https://www.pib.gov.in/PressReleasePag
e.aspx?PRID=1893205#:~:text=AMPHEX% 128. 'Prabhala Theertham, often seen in the news
20is%20aimed%20at%20joint,ever%20A during the festive season, is associated with
MPHEX%20conducted%20till%20date. which of the following regions in Andhra
Pradesh?
127. “The Citizen Perception Survey” is conducted
by? (1) Srikakulam (2) Konaseema
(1) Ministry of Housing and Urban Affairs, (3) Kurnool (4) Nellore
Government of India Answer: 2
(2) Ministry of Housing and Urban Affairs, Explanation:
Andhra Pradesh • "The tableau of Andhra Pradesh depicts
(3) Ministry of Personnel Public Grievances 'Prabhala Theertham'- a festival of the
and Pensions, Government of India peasantry during Makara Sankranti, at the
(4) The National Consumer Disputes Republic Day parade."
Redressal Commission • A 450-year-old tradition and culture of
Answer: 1 peasantry is protected religiously and is

AMIGOS IAS 44
followed by the residents of all villages in per year per family. Of this amount Rs.
the Konaseema district. The peasantry 6,000 will be paid from PM KISAN
bound with age-old customs and SCHEME.
traditions boasts of rich culture and • Financial Assistance is also being provided
heritage. to SC, ST, BC, Minority Category Landless
• Sankranti is one of the main festivals for tenant farmers and ROFR cultivators.
the people of Andhra Pradesh, which (ROFR - Recognition of Forest Rights).
comes after the harvest season. Prabhala • The amount will be paid in 3 instalments
Theertam festival entails taking out idols (not equal instalments) as detailed below.
built of bamboo frames and decorated Instalment Financial Assistance:
with colorful clothes, papers, feathers and • May Rs. 7,500/- (P.M. Kisan Assistance Rs.
Kathirkula (rice sheaves woven together 2000/-)
signifying prosperity).
• October Rs. 4,000/- (P.M. Kisan Assistance
• The tableau showcased how these Rs. 2000/-)
decorated bamboo arches with Lord
• January Rs. 2,000/- (exclusively of P.M.
Shiva's idols on them are taken around in
Kisan Assistance Rs. 2000/-)
a huge procession from all villages. People
• In 2021-22 the total beneficiaries 52.38
throng to the Prabhala Theertham and
lakh families of this 1.51 lakh families
enjoy the celebration amid the bursting of
comes under tenant farmers category.
firecrackers amid the playing of traditional
musical instruments and Garaga folk Source:
dance, which was also on display on the https://www.deccanchronicle.com/natio
tableau. n/in-other-news/310823/jagan-to-
release-rs-10974-crore-under-ysr-rythu-
Source :
bharosa-to-146324-t.html
https://timesofindia.indiatimes.com/city/
amaravati/state-to-showcase-prabhala- 130. Which of the following statements is correct
teertham-during-r-day- with respect to Dr YSR Abhaya Hastham
parade/articleshow/96926505.cms scheme?
129. Which the following statements is correct I. It is a contributory cum insurance scheme.
with respect to YSR Rythu Bharosa scheme? II. It provides lifelong pension of 1000/-per
I. This Financial Assistance is also being month to SHG Members.
provided to SC, ST, BC, Minority Category III. It provides the life insurance cover for the
Landless tenant farmers. age group of 18 to 59 years.
II. The amount is paid in 3 equal instalments. Select the correct answer from the options
III. Of this scheme amount, Rs. 6,000 is paid given below.
from PM KISAN Scheme. (1) Only I and II (2) Only II and III
Select the correct answer using the codes (3) Only I and III (4) All the above
given below. Answer: 3
(1) Only I and II (2) Only II and III Explanation:
(3) Only I and III (4) All the above
Answer: 3
Explanation:
Investment Support to Farmer - YSR Rythu
Bharosa :
• Dr. YSR Rythu Bharosa - PM Kisan : This
scheme was launched on 15th October
2019.
• Under this scheme, Financial Assistance is
being provided to Agriculture Land
Owning farmer families @ of Rs. 13,500

AMIGOS IAS 45
Source: • Girish Chandra Murmu, who is currently
https://www.thehindu.com/news/nation the Comptroller and Auditor General of
al/telangana/abhaya-hastam-deposit- India (CAG), has been selected as the
money-to-be-returned-to-shg- External Auditor of the International
women/article65218261.ece Labour Organization (ILO) in Geneva for a
131. Which of the following is the correct four-year term from 2024 to 2027.
arrangement of these places often seen in • CAG will take over from the incumbent
the news, from south to north on a map? External Auditor of ILO, Supreme Audit
(1) Gaza Strip – Golan Heights – West Bank Institution of Philippines.
(2) Gaza Strip – West Bank – Golan Heights 134. The union Home minister recently visited
(3) Golan Heights – Gaza Strip – West Bank “Harami Nala” and Border Observation Posts
(4) Golan Heights – West Bank – Gaza Strip (BOPs) of the Border Security Force (BSF).
Where is “Harami Nala” located?
Answer: 2
(1) Pak Occupied Kashmir (PoK)
Explanation:
(2) Sir Creek Area of Kutch
(3) Andaman Sea
(4) Arunachal Pradesh
Answer: 2
Explanation:
Harami Nala:
• The union Home minister visited “Harami
Nala” and Border Observation Posts
(BOPs) of the Border Security Force (BSF)
in the Sir Creek area of Kutch, Gujarat,
along the India-Pakistan international
 Golan Heights – West Bank – Gaza Strip is
border.
the correct arrangement of these places
from North to South on a map. • Harami Nala is known for its inhospitable
conditions and has been a hotspot for
132. Who won the 2023 Booker Prize for Fiction
cross-border crime and exploitation by
for his or her novel ‘Prophet Song’?
infiltrators.
(1) Geetanjali Shree (2) Margaret Atwood
About Harami Nala:
(3) Salman Rushdie (4) Paul Lynch
• Harami Nala is a natural water body
Answer: 4
approximately 25 kilometres long that
Explanation: flows west to east from Pakistan into
Irish author Paul Lynch won the 2023 Booker Kutch, south of the Vighakot sector.
Prize for his novel ‘Prophet Song’, which is • The channel is flanked by vast marshy
a dystopian work set in an Ireland that mudflats that are affected by low and high
descends into tyranny. tides.
133. Who has been selected as the External • About 22 odd kilometres of the channel lie
Auditor of the International Labour on the Indian side of the “vertical line” of
Organization (ILO) for a four-year term from the IB between India and Pakistan, which
2024 to 2027? is not contested.
(1) Praveen Mehta • It is extremely rich in fishing, especially
(2) Girish Chandra Murmu prawns.
(3) RG Vishwanathan • Pakistani fishing boats try to enter the
(4) Aditya Roy Chowdary Harami Nala on the Indian side, drawn by
Answer: 2 the possibilities of better fish catch.
Explanation: • The Indian side is mostly empty over the
entire spread of the Harami Nala except

AMIGOS IAS 46
for factories producing salt, bromide, and differently abled categories will be
other chemicals. reimbursed.
o Jagananna Vidya Deevena (RTF): to
provide full fee reimbursement to every
eligible student, i.e. Full Fee (Tuition Fee,
Special Fees,Other Fees & Exam Fees) as
defined in the G.O.Ms.No.66, SW(Edn)
Dept., dated 8-9-2010 and as fixed by the
competent authorities is reimbursed to all
eligible students. ii. Full fee shall be
credited to the respective College
accounts on behalf of students.
135. Which two countries are conducting the o Jagananna Vasathi Deevena (MTF): to
naval exercise "Sea Guardian-3" in the provide Rs.10,000/- per person to ITI
Northern Arabian Sea? students, Rs.15,000/- per person to
Polytechnic students.Rs.20,000/- per
(1) China and Pakistan
person for other Degree and above
(2) Russia and Iran
courses per year to every eligible student
(3) Japan and Philippines for food and hostel expenses.
(4) India and Oman JaganannaVasathi Deevena shall be
Answer: 1 credited into the respective account of the
Explanation: mother of the eligible student. In case of
 The naval exercise ‘Sea Guardian-3’ is the demise or absence of the mother, the
being conducted in the Northern Arabian amount shall be credited to the account of
Sea by China and Pakistan. This is the 3rd the natural guardian of the student.
edition of the bilateral naval drill between • 75% aggregate attendance is mandatory
the two countries. for the release of scholarship.
136. Which of the following statement is correct • Any student whose annual family income
with reference to Jagananna Vidhya Deevena is less than Rs. 2.5 lakh are eligible under
and Vasati Deevena Schemes? Jagananna Vidya Deevena Scheme.
I. Vasati Deevena provides Rs.10,000/- per • Those with 10 acres of wetland and 25
person to ITI students, Rs.15,000/- per acres of dry land are also eligible for the
person to Polytechnic students and benefit.
Rs.20,000/- per person for other Degree • There is no income limit for students
and above courses per year. hailing from families of sanitation works
II. Any student whose annual family income and those who are professionally
is less than Rs. 1.5 lakh are eligible under dependent on taxis, auto, and tractor.
Jagananna Vidya Deevena Scheme. However, the income taxpayers will not be
Select the correct answer from the options eligible.
given below. • Initially, the full fee reimbursement will be
(1) Only I (2) Only II extended to students pursuing B Tech, B
(3) Both I and II (4) Neither I nor 2 Pharmacy, M Tech, M Pharmacy, MBA,
Answer: 1 MCA, BEd and such courses
Explanation: In - eligible Students:
• Under Jagan Anna Vidya Deevena Scheme, The students under the following categories
the fee of students belonging to are not eligible for availing the Schemes.
Scheduled Castes, Scheduled Tribes, i. Studying in Private Universities / Deemed
backward classes, minorities, Kapus, Universities.
economically backward classes and ii. Pursuing Correspondence / Distance
education courses.

AMIGOS IAS 47
iii. Admitted under Management / Sports (3) Both I and II (4) Neither I Nor II
Quota. Answer: 3
Source: Explanation:
https://www.thehindu.com/news/nation • The Union government's 'sponge cities'
al/andhra-pradesh/jsp-alleges- mission will cover 15 urban local bodies of
misappropriation-of-120-crore-in-vidya- Andhra Pradesh, including
kanuka-scheme/article67532545.ece Visakhapatnam, Rajahmundry, Kakinada,
137. Consider the following statements with Guntur, Kadapa, Eluru, Bhimavaram,
respect to the Jagananna Civil Services Ongole, Adoni, etc.
Prothsahakam. • A sponge city is a city that is designed to
I. It is to provide financial assistance as cash passively absorb, clean, and use rainfall in
incentive to the aspirants of socially, an ecologically friendly way that reduces
educationally and economically dangerous and polluted runoff.
weaker/backward sections. • There is a lack of a proper drainage
II. It provides Rs.1 lakh and Rs.50,000 who network in the cities.
qualified in the Preliminary and Mains Civil • When we build cities, we build on
Services Examination conducted by the wetlands and ponds, which actually have
UPSC respectively. the ability to soak in extra water.
Select the correct answer. • Cities are getting bigger, and climate
(1) I Only (2) II Only change is threatening to bring more
(3) Both I and II (4) Neither I Nor II extreme weather events.
Answer: 3 Source:
Explanation: https://www.thehindu.com/opinion/lead
• Government sanction new scheme /time-for-a-sponge-cities-mission-in-
Jagananna Civil Services Prothsahakam to india/article32985979.ece
provide financial assistance as cash 139. Which of the following statements is correct
incentive to theaspirants of socially, with reference to “YSR Kalayana Masthu
educationally and economically scheme”?
weaker/backward sections of Andhra I. The bride must have passed Class 10th
Pradesh, who qualified in the Preliminary and be at least 18 years old at the time of
and Mains Civil Services Examination marriage.
conducted by the UPSC for an amount of II. Brides belonging to construction worker
Rs.1 lakh and Rs.50,000 respectively in families are also eligible for this scheme.
order to increase the representation of Select the correct answer using the codes
A.P in the Civil Services. given below.
Source: (1) I Only (2) II Only
https://www.thehindu.com/news/nation (3) Both I and II (4) Neither I Nor II
al/andhra-pradesh/andhra-pradesh-
Answer: 3
cabinet-gives-nod-to-jagananna-civil-
Explanation:
services-incentive-
scheme/article67327198.ece
138. Consider the following statements with
reference to “sponge cities mission.”
I. A sponge city is a city that is designed to
passively absorb, clean, and use rainfall in
an ecologically friendly way.
II. It will cover 15 urban local bodies in
Andhra Pradesh.
Select the correct answer.
(1) I Only (2) II Only

AMIGOS IAS 48
(4) Genetic engineering
Answer:3
Explanation:
 The Bletchley Declaration on responsible
development of Artificial Intelligence was
endorsed by 28 countries and the
European Union. It encourages
transparency and accountability from AI
developers on monitoring and mitigating
potentially harmful capabilities.
142. “MONDIACULT 2022 World Conference” aims
to?
(1) talk about sustainable development and
cultural policies.
Source: (2) encourage nuclear science and technology
https://www.thehindu.com/news/nation (3) highlight scientific and technological
al/andhra-pradesh/andhra-pradesh-govt- developments
releases-8164-crore-for-ysr-
(4) addressing global issues in the defence
kalyanamasthu-shadi-tohfa-
industry
beneficiaries/article67565398.ece
Answer: 1
140. The Fourth World Kuchipudi Day was
Explanation:
celebrated on?
 The UNESCO-MONDIACULT 2022 World
(1) October 20 (2) October 15
Conference focused on cultural policies
(3) October 7 (4) October 25
and sustainable development, aiming to
Answer: 2 shape a more robust and resilient cultural
Explanation: sector.
• Fourth World Kuchipudi Day was 143. Which of the following countries is a member
celebrated on October 15, 2023. of the Arab League??
• Kuchipudi, one of the major dance forms I. Syria
of India, originated in Andhra Pradesh. II. Saudi Arabia
Kuchipudi is the state classical dance of
III. Israel
Andhra Pradesh.
Select the correct answer from the options
• Kuchipudi derives its name from the
given below.
village of Kuchelapuram, where it was
(1) Only I (2) Only II
nurtured by great scholars and artists who
refined the dance technique. (3) Only I and II (4) All I, II, III
• It grew largely as a product of the Bhakti Answer: 3
movement beginning in the 7th century Explanation:
AD. • The Arab League re-admitted the Syrian
Sources: government, ending a more than decade-
https://www.thehansindia.com/hans/you long suspension and isolation.
ng-hans/world-kuchipudi-day-celebrated • In November 2011, the Arab League
141. “The Bletchley Declaration," was recently suspended Syria over its crackdown on
seen in the news, aims to promote peaceful protests that took the shape of a
responsible development of which civil conflict that led to the killing of over
technology? 5,00,000 people.
(1) Nuclear technology • After a 12-year suspension, Arab
(2) Space technology government delegates voted to reinstate
Syria in the Arab League in 2023.
(3) Artificial intelligence

AMIGOS IAS 49
• The Arab League is a regional organisation Select the correct answer from the options
of Arab states in the Middle East. given below.
• It was formed in Cairo in 1945 after a pact (1) Only I, II, III (2) Only II, III, IV
of the League was signed by six members, (3) Only I, III,IV (4) All the above
namely Egypt, Iraq, Jordan, Lebanon, Answer:1
Saudi Arabia, and Syria. Explanation:
• The Arab League has 22 member states, • ADB is a multilateral development finance
which include Algeria, Bahrain, Comoros, institution.
Djibouti, Egypt, Iraq, Jordan, Kuwait,
• It provides finance to African governments
Lebanon, Libya, Mauritania, Morocco,
and private companies investing in the
Oman, the Palestinian Authority, Qatar,
Regional Member Countries (RMC).
Saudi Arabia, Somalia, Sudan, Syria,
• It was founded in 1948.
Tunisia, the United Arab Emirates, and
Yemen. • Its fundamental duty is to fight poverty
and improve the standard of living of the
• Israel is not a member of the Arab League.
continent by investing in projects and
144. Consider the following statements:
programs which uplifts the socio-
I. Moscow Format is a dialogue platform on economic conditions of the common
the peace process in Afghanistan. mass.
II. India is not a party to the Moscow Format. • The Headquarter is located in in Abidjan,
Which of the statements given above is/are Côte d’Ivoire. But it moved to Tunis,
correct? Tunisia in 2003, before returning in
(1) Only I (2) Only II September 2014.
(3) Both I and II (4) Neither I Nor II • The AfDB is controlled by a Board of
Answer: 1 Executive Directors, made up of
Explanation: representatives of its member countries.
About Moscow Format: • Member governments are officially
• The Moscow format is one of the several represented at the AfDB by their Minister
dialogue platforms on Afghanistan, which of Finance, Planning or Cooperation who
began before the Taliban takeover of sits on the AfDB Board of Governors.
Kabul. • It is to be noted that Cabo Verde and Libya
• The key objective of the Moscow format are the only two countries are without the
of consultations is to facilitate the national beneficiary of AfDB or ADF.
reconciliation process in Afghanistan and • The AfDB comprises three entities: the
establish peace. African Development Bank (ADB), the
• A dialogue platform on Afghanistan, African Development Fund (ADF) and the
initiated in 2017. Nigeria Trust Fund (NTF).
• To promote regional peace and 146. Consider the following statements with
cooperation. respect to YSR Law Nestham.
• 6 founding members; now 9 members: I. The name of the applicant shall be entered
China, India, Iran, Kazakhstan, Kyrgyzstan, in the rolls of advocates being maintained
Pakistan, Russia, Turkmenistan, and by the Andhra Pradesh State Bar Council.
Uzbekistan. II. It provides a monthly stipend of ₹ 3,000/-
145. African Development Bank (AfDB) Group for the first three years
comprises of how many of these entities? III. Non-practicing advocates are also eligible
I. African Development Bank (ADB) for the scheme to encourage them to
II. African Development Fund (ADF) practice.
III. Nigeria Trust Fund (NTF) Which of the above statements is/are not
correct?
IV. African Union

AMIGOS IAS 50
(1) Only I and II (2) Only II and III (1) Consumers
(3) Only I and III (4) All the above (2) Students
Answer: 2 (3) Government functionaries
Explanation: (4) Citizens
• This scheme was launched in 2019. Answer: 2
• This scheme is being implemented by the Explanation:
Law Department, Government of Andhra • The Government of Andhra Pradesh is set
Pradesh. to introduce a generative artificial
• Benefit: A monthly stipend of an amount intelligence (AI) bot for students studying
of ₹ 5,000/- for the first three years. in government schools.
Eligibility: • The bot called "Doubt Clearance Bot,"
• The applicant should be a which was developed by ConveGenius,
resident/domicile of the state of Andhra will be available to students 24/7 to
Pradesh. answer their doubts and queries.
• The applicant shall possess bachelor’s • It will answer queries in the subjects of
degree in law. physics, chemistry, biology, mathematics,
• The applicant should be a Law Graduate, English, and TOEFL preparation.
who passed out in the year 2016 and • The bot will be installed on all the tablets
afterward. and Interactive Flat Panels (IFPs) supplied
• The name of the applicant shall be entered to teachers and students.
in the rolls of Advocates being maintained • The Doubt Clearance Bot is part of the
by the Andhra Pradesh State Bar Council Andhra Pradesh government's
under Section 17 of the Advocates Act, commitment to providing high-quality
1961. education to students and preparing them
• The Junior Advocates, who started for the future.
practice and have not crossed the first Source:
three (3) years of practice, as of the date https://www.deccanchronicle.com/natio
of issue of the Government Order, are n/in-other-news/191023/andhra-
eligible for a stipend for the remaining pradesh-to-introduce-generative-ai-bot-
period. for-students.html
Exclusions: 148. Which of the following statements is/are
• The Junior Advocates, who have crossed correct with respect to “Jaganannaku
the first three (3) years of practice, as of Chebudham Program”.
the date of issue of the G.O., are not I. It replaced the existing “ Spandana”
eligible. grievance redressal program.
• The Applicant possessing four-wheelers in II. Citizens can use this helpline by calling
his name is not eligible. 1902 to register individual or household-
• A Non - Practicing Advocates are not level grievances.
eligible. Select the correct answer using the codes
Source: https://news.abplive.com/andhra- given below.
pradesh/andhra-pradesh-cm-ys-jagan- (1) I Only
mohan-reddy-releases-ysr-law-nestham- (2) II Only
funds-for-over-2-000-lawyers-1583822 (3) Both I and II
147. The Government of Andhra Pradesh is set to (4) Neither I Nor II
introduce a generative artificial intelligence Answer: 2
(AI) bot called “Doubt Clearance Bot." For Explanation:
whom is it targeted to clear the doubts?
• Chief Minister Y.S. Jagan Mohan Reddy
started an innovative Jaganannaku
Chebudam' programme to listen to the

AMIGOS IAS 51
grievances from the public and resolve 150. Consider the following statements with
them possibly on-the-spot. respect to the Jagananna Saswatha
• Jaganannaku Chebudham is a universal Bhuhakku-Bhuraksha scheme:
grievance redressal helpline that allows I. A comprehensive resurvey programme
citizens to directly raise their issue to the will be launched in order to store the
Chief Minister’s Office. digital records of land.
• As a part of the initiatives, the state II. The resurvey project resurvey properties
government has launched the new in only rural Areas.
outreach programme is an improvised Select the correct answer.
version of the Spandana. It is (1) I Only
improvisation of the existing Spandana (2) II Only
programme.
(3) Both I and II
• Citizens can use this helpline by calling
(4) Neither I Nor II
1902 to ✓ Register individual or
Answer: 1
household-level grievances ✓ Follow up
Explanation:
on the status of a registered grievance ✓
Jagananna Bhuhakku and Bhuraksha Scheme:
Enquire about government services and
• The Chief Minister of Andhra Pradesh
schemes
asked officials to complete the third phase
Source:
of the Jagananna Saswatha Bhuhakku-
https://www.deccanchronicle.com/natio
Bhuraksha by January 2024. Jagananna
n/current-affairs/040223/cm-to-start-
Bhuhakku and Bhuraksha Scheme.
jaganannaku-chebudam-grievance-
• The Chief Minister of Andhra Pradesh
programme.html
launched Jagananna Saswatha Bhuhakku
149. Recently, Andhra Pradesh Chief Minister
Bhuraksha Scheme on 18th January 2022.
Jagan Mohan Reddy launched the “INDGAP
• Through this scheme, a comprehensive
programme.” What is the aim of this
resurvey programme will be launched in
programme?
order to store the digital records of land.
(1) to bridge the gaps in the education
Hence, Statement I is correct.
system.
• The registration of lands in ward/village
(2) to bridge the gaps in the healthcare
secretariat will commence soon after the
system.
completion of the comprehensive
(3) to help the government facilitate export of resurvey.
farm yield at a premium price.
• The resurvey project includes properties
(4) To bridge the gaps in public service in urban areas apart from Rural Areas.
delivery. Hence, Statement II is not correct.
Answer: 3 • The ultimate goal of the resurvey is to
Explanation: create a database of all immovable
• Andhra Pradesh Chief Minister Jagan properties that serves as a conclusive
Mohan Reddy launched “INDGAP record of titles as per S&B (AP Survey and
programme.” The certification will help Boundaries) act 1923.
the government facilitate export of farm Sources:
yield at a premium price not only https://www.thehindu.com/news/nation
domestically but also to more than one al/andhra-pradesh/cabinet-panel-asks-
hundred countries, including Europe and officials-to-complete-third-phase-of-land-
U.S. resurvey-work-by-
https://www.thehindu.com/news/national/a january/article67430977.ece
ndhra-pradesh/andhra-pradesh-chief-
minister-jagan-mohan-reddy-launches-
indgap-programme/article66785683.ece

AMIGOS IAS 52

You might also like